UWorld Review- Pediatrics

Pataasin ang iyong marka sa homework at exams ngayon gamit ang Quizwiz!

A. Cytomegalovirus Cytomegalovirus infection is the m/c congenital infection & classically presents with jaundice, petechiae & hepatosplenomegaly, as well as periventricular calcifications and microcephaly.

1-day-old girl is evaluated for petechiae and jaundice. She was born at 36 weeks gestation to a 24-year-old woman, gravida 3 para 3, who had no prenatal care. During the second trimester, the mother had a week of fever & sore throat, which resolved without treatment. Maternal lab studies obtain on admission revealed a negative rapid plasma reagin test. Weight and length are at the 10th percentile, and head circumference is at < 3rd percentile. Temp is 36.9C (98.4). PE shows a diffuse petechial rash, jaundice of the face and trunk, and hepatosplenomegaly. Head imaging shows periventricular calcifications. Which of the following is the most likely cause of this infant's symptoms? A. Cytomegalovirus B. HSV C. Listeria moncytogenes D. Parvovirus B19 E. Treponema pallidum

A. Abusive head trauma Abusive head trauma can be caused by vigorous shaking of an infant & results in subdural hemorrhage (due to tearing of bridging veins) & retinal hemorrhages. Additional red flags are injuries inconsistent with history or developmental age.

1-month-old, full-term boy is brought to the ED by his mother due to difficulty awakening him. The mother says that the infant was left alone for a few mins this morning & rolled off a bed. Since then, he has been sleepy and less active than usual. Vitals are normal. PE shows a lethargic boy with a large, full anterior fontanelle. Bilateral retinal hemorrhages are seen on funduscopic exam. Which of the following should be strongly suspected in this patient? A. Abusive head trauma B. Bacterial meningitis C. Idiopathic thrombocytopenia purpura D. Neonatal vitamin K deficiency E. Unintentional head injury

A. Acute lymphoblastic leukemia ALL is m/c seen in boys age 2-5 & presents with anemia, thrombocytopenia & a normal or elevated WBC count. The presence of blasts on peripheral smear & bone marrow biopsy is characteristic.

2-year-old boy is brought by his mother to the ED due to a high-grade fever that is resistant to acetaminophen. For the last 4 days, the child has been very irritable. He has been coughing frequently & has a poor appetite. Temp is 38.8C (102.2F), BP is 90/60, HR is 119/min. Exam reveals enlarged cervical lymph nodes & a liver edge that is 2.5 cm below the costal margin. A spleen tip is also palpable. Scattered bruises are present throughout the trunk. Lab results are as follows: CBC- Hemoglobin: 7.0 g/dL (normal: 13.5-17.5) Platelets: 5,000/mm3 (normal: 150,000-400,000) Leukocytes- Blasts: 90% (normal: Lymphocytes: 10% (normal: 25-33%) CXR is normal. Which of the following is the most likely diagnosis in this patient? A. Acute lymphoblastic leukemia B. Burkitt lymphoma C. Chronic lymphocytic leukemia D. Hodgkin lymphoma E. Multiple myeloma

B. Hyperpronation of the right forearm Subluxation of the radial head most commmonly occurs in children age 1-4 from axial traction on the forearm with the elbow in extension. Subluxation can be reduced with hyperpronation of supination followed by flexion of the elbow, although hyperpronation is the preferred method on the first attempt

A previously healthy 11-month-old child is brought to the ED with a right arm injury. Her mother tied to lift her up out of the stroller by holding onto her right hand while she was still buckled in. The infant started crying immediately & has been refusing to use the right arm. On exam, the child is alert & playful. When shown an object, the infant will grasp with the left, but not the right hand. She holds the right arm in extension with the forearm pronated. There is no tenderness to palpation of the humerus or forearm. No swelling or deformity is noted. Which of the following is the best next step in management of this patient? A. Anteroposterior & lateral XR of the right elbow B. Hyperpronation of the right forearm C. Referral to orthopedics for surgery D. Supination & hyperextension of the right forearm E. Supportive management with a sling & pain control

D. Perform surgical correction at age 6 months Isolated hypospadias, a congenital male anomaly presenting with a ventrally positioned urethral meatus, is managed with surgical correction at age 6 months.

1-day-old boy is being evaluated routinely in the nursery He was born to a 37-year-old primigravida at 38 weeks gestation via vaginal delivery. The mother received routine prenatal care & is healthy On exam of the boy's external genitalia, a ventrally positioned urethral meatus is noted on the shaft of the penis & the penis is curved downward. Both testicles are easily palpated within the scrotum. The remainder of the exam is normal. What is the best next step in management of this patient's condition? A. Obtain karyotype analysis B. Obtain pelvic & renal US C. Perform circumcision without delay D. Perform surgical correction at age 6 months E. Provided reassurance that it will resolve as the penis enlarges

C. Prostaglandin E1 As the patient's ductus arteriosus closes, ductal-dependent congenital heart defect can present with cyanosis & hypoxia. Prostaglandin E1 maintains the latency of the ductus arteriosus & is lifesaving in infants with ducal-dependent congenital heart defects. This patient's presentation of sudden clinical deterioration after an initial normal period suggests that the deterioration is due to patent ductus arteriosus (PDA) closure.

1-day-old boy is brought to the ED due to cyanosis that has worsened. He was delivered at home yesterday via spontaneous vaginal delivery. The mother received no prenatal care & is unsure of the gestational age at delivery. The neonate did well after delivery & has breastfed several times. 2 hours ago, he developed rapid breathing & cyanosis. Pulse ox from the right hand is 80% on room air & does not improve with 100% inspired oxygen. PE shows a diffusely cyanosis infant with hypotension, tachycardia & tachypnea. CXR is normal. Labs show a hematocrit of 60% (normal: 47-65%). Echocardiogram is pending. Which of the following is the best next step in the management of this patient? A. Indomethacin B. partial exchange transfusion C. Prostaglandin E1 D. Sildenafil E. Surfactant

B. Cephalohematoma Cephalohematoma is a subperiosteal hemorrhage that presents in the 1st day of life with a firm, well-demarcated scalp swelling that does not cross suture lines. Forceps- or vacuum-assisted deliveries increase the risk of developing cephalohematomas. Management is with observation b/c most resorb spontaneously.

1-day-old boy is evaluated in the neonatal ICU. The patient was born at 34 weeks gestation via forceps-assisted vaginal delivery. APGAR scores were 5 and 8 at 1 and 5 mins, respectively. Exam shows a firm, well-demarcated swelling on the right parietal scalp with no discoloration or apparent tenderness. Which of the following is the most likely cause of this patient's physical exam findings? A. Caput succedaneum B. Cephalohematoma C. Hydrocephalus D. Plagiocepalus E. Subgaleal hemorrhage

B. Echocardiography The classic ventricular septal defect murmur is a harsh, holosystolic murmur best heard at the left lower sternal border. Echo should be performed to determine the location & side of defect & to rule out other defects. Most small ventricular septal defects close spontaneously & require no treatment.

1-month-old boy is brought to the office for a routine visit. His prenatal & birth histories are unremarkable. His vitals and pulse ox are normal. Exam shows a harsh, 3/6 holosystolic murmur over the left lower sternal border. Palpation indicates no thrill over the precordial region. Brachial & femoral pulses are 2+ and equal. The skin is pin & well perfused. ECG is normal. Which of the following is the best next step in management of this patient? A. Digoxin & diuretic therapy B. Echocardiography C. Oxygen administration D. Prostaglandin E1 therapy E. Reassurance & no further workup

D. Intussusception Intussusception (telescoping bowel) is most common in children age 6-36 months and cause episodic abdominal pain, currant jelly stools and lethargy. A palpable, sausage-shaped abdominal mass is not always present. This is an intestinal disorder in which one segment of bowel telescopes into the lumen of another. The telescoped bowel appears as a target sign on US and air or saline enema is both diagnostic and therapeutic. Surgery is indicated if the enema does not reduce the intussusception of if imaging suggests a mass lesion.

10-month-old girl is brought to the ED due to bright red stools. About an hour ago, the father found bright red-tinged, formed stools mixed with mucus in the patient's diaper. The patient has had intermittent abdominal pain since yesterday. These episodes resolve after 20-25 mins but have been occurring more frequently since this morning. The patient goes to day care and several other children have had diarrhea over the past week. Temp is 27.3C (99.1F). Exam reveals diffuse abdominal tenderness, the patient also appears tired. Fecal testing is positive for blood & negative for leukocytes. Which of the following is the most likely diagnosis in this patient? A. Acute dysentery B. Appendicitis C. Crohns disease D. Intussusception E. Juvenile polyps

B. Clothing removal Pediatric caustic ingestions are often accompanied by exposure of the skin to the caustic agent. Contaminated clothing should be removed promptly to avoid further injury.

18-month-old boy is brought to the ED an hour after drinking liquid oven cleaner from an unlocked kitchen cabinet. His parents tried to give him water & milk but he has difficulty swallowing. The boy also has blood-tinged oral secretions. VS are stable. Exam shows an anxious child who is crying & drooling. His lips & chin are swollen & erythematous. He has no strider & his breathing pattern appears normal. Lungs are CTA. His shirt & pants are covered in oven cleaner. Which of the following is the best next step in management of this patient? A. Barium swallow study B. Clothing removal C. NG feeding tube D. Neutralization with vinegar E. Upper GI endoscopy

A. Ewing sarcoma Ewing sarcomas are malignant tumors that occur most commonly in the pelvis & long bones of white, adolescent boys. Localized pain & swelling can be accompanied by systemic findings & characteristic "onion skinning" (lamellated periosteal retraction) & "moth-eaten" appearance on XR

10-year-old boy is brought in due to worsening left thigh pain & difficulty walking for several months. The pain is continuous, no radiating & is worse at night. Ibuprofen has provided no relief. He has had occasional fever over the last month; temp: 37.2C (99F). The patient has an antalgic gait & favors the right side. the left thigh is swollen & erythematous & has a palpable tender, immobile mass. XR shows a central lytic lesion in the diaphysis of the femur with cortical layering, a "moth-eaten" appearance & extension into the soft tissue. Which of the following is the most likely diagnosis in this patient? A. Ewing sarcoma B. Giant cell tumor of bone C. Juvenile idiopathic arthritis D. Multiple myeloma E. Osteoporosis osteoma

B. Prevent acute rheumatic fever The primary goal of treating strep pharyngitis with a 10-day course of oral PCN or Amoxicillin is to prevent acute rheumatic fever. The role of ABXs in the prevention of poststreptococcal glomerulonephritis is unclear.

10-year-old boy is brought in for a follow up exam. He was diagnosed with strep pharyngitis 5 days ago after developing odynophagia, tonsillitis enlargement & posterior pharyngeal exudates. The patient is on day 5 of a 10-day course or oral PCN & feels much better. He has no chronic medical conditions & no allergies. VS & PE are normal. The parents ask if the ABX can be discontinued. Which of the following is the rationale for continued ABX therapy in this patient? A. Minimize spread of infection B. Prevent acute rheumatic fever C. Prevent invasive pneumonia D. Prevent peritonsillar abscess E. Prevent poststreptococcal glomerulonephritis

C. Doxycycline Rocky Mountain spotted fever is a tick-borne illness characterized by fever, HA & an erythematous macular &/or petechial rash on the wrists & ankles. Prompt, empiric administration of Doxycycline is the treatment of choice.

10-year-old boy is brought to the ED due to low-grade fever over the last 3 days. For the last 2 days, the patient has also had a diffuse HA that is now severe & associated with nausea. Today he is weak & has diffuse myalgia & arthralgia. The patient returned from a family camping trip in North Carolina 1 week ago. Temp is 38.8C (101.8F). The patient appears lethargic. Oropharyngeal mucosa is normal. Cardiac exam is normal. Skin exam shows a macular, erythematous rash & few petechiae on bilateral wrists & ankles. Kernig & Brudzinski signs are both negative. Leukocytes are 5,800/mm3 & platelets are 90,000/mm3. Which of the following is the best therapy option for this patient? A. Amoxicillin B. Ceftriaxone C. Doxycycline D. IV immunoglobulin E. No therapy is indicated

A. oral ABX clinical diagnosis of AOM requires otoscopic evidence of a middle ear effusion & signs of acute inflammation or a bulging TM. Oral ABX (high-dose Amoxicillin) are administered to children age < 6 months & those > 6 months with high fever, severe pain or bilateral disease.

11-month-old girl is brought in due to 3 days of fever with rhinorrhea & nasal congestion for the past week. Her mother reports she has been grabbing & reaching for her left ear for the last 2 days. The patient has no chronic medical conditions. She takes no meds regularly but has recently completed a course of oral ABX for an ear infection 6 weeks ago. The patient attends daycare 4 days/week. Both parents smoke cigarettes & both older siblings required tympanovstomy tubes as infants. Temp is 39.4C (102.9F). The patient is irritable but easily consoled. Otoscopy shows bilateral bulging & pink TM with poor mobility on insufflation. Crusted rhinorrhea is present at the nares. The remainder of the exam is unremarkable. What is the next best step in management of this patient? A. oral ABX B. ototopical ABX C. supportive care & observation D. tympanocentesis & culture E. tympanovstomy tube placement

C. Aplastic crisis Patients with sickle cell disease can develop aplastic crisis characterized by an acute drop in hemoglobin, a reticulocyte count < 1% & no splenomegaly. Parvovirus B19 is the most common cause

11-year-old boy with sickle cell disease is brought to the ED with worsening SOB, weakness & fatigue over the past 3 days. The patient has no fever, cough, or chest pain. He has had many hospitalizations for acute pain crises & acute chest syndrome. Temp is 36.7C (98F), Pulse Ox on room air: 98%. PE shows a pale, tired-appearing boy. Cardiopulmonary exam is normal. The abdomen is soft with no palpable liver or spleen. Labs reveal markedly reduced hemoglobin level & reticulocyte count; leukocytes & platelets are normal. Which of the following is the most likely cause of this patient's symptoms? A. Acute chest syndrome B. Aplastic anemia C. Aplastic crisis D. Myocardial infarction E. Splenic sequestration crisis

C. Begin ABX therapy In patients with juvenile idiopathic arthritis who have an atypical flare of a single joint, such as severe pain with nighttime awakening, septic arthritis should be considered. An elevated synovial leukocyte count >50,000/mm3 with a neutrophil predominance supports the diagnosis. Treatment is immediate ABX therapy.

11-year-old girl is brought to the ED due to right shoulder pain. The patient first noticed the pain 4 days ago. The pain has gradually increased, awakens her at night and today she is unable to lift her right arm to brush her teeth. 4 months ago, the patient was diagnosed with polyarticular juvenile idiopathic arthritis for which she takes Methotrexate. Temp is 38.5C (101.3F). The patient hold her right arm at her side & resists attempts to move the shoulder due to pain. The shoulder joint is aspirated, and synovial fluid analysis reveals leukocyte count of 55,000 cells/mm3, with 80% polymorphonuclear cells. Which of the following is the best next step in management of this patient? A. Add a biologic disease-modifying anti rheumatic drug B. Administer intra-articular glucocorticoids C. Begin ABX therapy D. Increase Methotrexate dose E. Recommend NSAIDs & rest

D. prophylactic Rifampin Rifampin is the most frequently used agent as chemoprophylaxis for meningococcal disease. in this case, patient has Neisseria meningitis which is transmitted by direct contact with contaminated respiratory secretions or aerosolized droplets.

12- year old boy brought to ED by paramedics after being found unresponsive at home. Patient had 24 hrs of HA, nausea & fever. Temp is 39.6C (103.3F). PE reveals neck stiffness. LP is performed and analysis of CSF shows marked pleocytosis and gram stain shows gram-negative cocci in pairs. Parents concerned about their younger son who shares a bedroom with patient. What is the most appropriate management of this patient's household contacts? A. immediate vaccination B. observation only C. prophylactic PCN D. prophylactic Rifampin E. prophylactic sulfamethoxazole

C. Normal platelets, normal PT, prolonged PTT Von Willebrand disease is the most common inherited bleeding disorder & is usually transmitted in an autosomal dominant fashion. Symptomatic patients typically present with easy bruising & mucosal bleeding. Lab eval shows normal platelets, normal PT & either normal or prolonged PTT.

12-month-old boy is brought in by his father due to prolonged bleeding from his mouth after slipping & hitting his face on a coffee table. He was born at term & his mother died shortly after his birth due to bleeding complications. VS are normal. There is blood oozing from his gums & marked bruises along his trunk & thigh. The remainder of the PE is normal. Lab eval shows decreased von Willebrand Factor antigen levels & activity. Which of the following sets of labs values is most likely present in this patient? A. Decreased platelets, normal PT, normal PTT B. Decreased platelets, prolonged PT, prolonged PTT C. Normal platelets, normal PT, prolonged PTT D. Normal platelets, prolonged PT, normal PTT E. Normal platelets, prolonged PT, prolonged PTT

B. G6PD deficiency is an X-linked disorder commonly seen in African American men. It is characterized by episodic hemolysis in response to oxidants, drugs, infections or certain foods (fava beans). Heinz bodies & bite cells are characteristically seen on peripheral blood smear

12-year-old black boy is brought to the ED with back pain, abdominal pain, & dark urine that began earlier this morning. The patient has never had symptoms like these before. 2 days ago, he developed fever, runny nose & cough. Temp: 37C (98.6F), BP: 110/60 & HR: 100/min. The patient appears tired & has scleral icterus. Cardiopulmonary exam is normal. Abdominal exam is normal. Abdominal exam reveals mild, diffuse tenderness with palpation but no rebound tenderness or rigidity. Hemoglobin is 8.2g/dL. Peripheral blood smear reveals bite cells & Heinz bodies. What is the most likely cause of this patient's condition? A. acute intermittent porphyria B. G6PD deficiency C. hereditary spherocytosis D. sickle cell disease E. thalassemia minor

E. Peripheral nerve fibers Guillain-Barre Syndrome is an immune-mediated polyneuropathy caused by demyelination of peripheral nerve fibers. Most patients have a preceding respiratory or GI infection (campylobacter), that triggers an initial immune response These antibodies are then redirected toward cross-reacting antigens on myelin or axons, a process known as molecular mimicry. Motor nerves are m/c affected, but sensory and autonomic nerves may also be involved. Presentation influxes ascending weakness (starts in lower extremities & ascends to involve the respiratory & bulbar muscles), often accompanied by paresthesias & neuropathic pain.

12-year-old boy is brought to the ED b/c "he cannot walk anymore." The patient has had a pins-and-needles sensation & weakness in both feet for the past few days. This morning, he could not stand or move his legs and has bilateral thigh pain. 2 weeks ago, the patient had a febrile diarrheal illness that resolved on its own. The patient's vaccinations are up to date. Temp is 36.7C (98.1F), BP is 150/90, HR is 92/min and RR are 16/min. PE shows symmetric flaccid paralysis of the lower extremities and absent ankle and patellar DTRs. Vibration sensation is intact in both legs. The upper extremities are weak and hyporeflexic. Which of the following structures is primarily impaired in this patient's condition? A. Anterior horn cells B. Dorsal and lateral spinal columns C. Muscle fibers D. Neuromuscular junction E. Peripheral nerve fibers

C. Shearing at the proximal femoral physis Slipped capital femoral epiphysis occurs when excessive shearing at the proximal femoral physis weakens the growth plate, causing displacement of the proximal femur diaphysis. Patients are classically obese adolescents with chronic, dull pain along the thigh or knee that worsens with activity.

12-year-old girl is brought to the clinic by her mother for eval of leg pain. The patient first noticed a dull ache in her left thigh a month ago. The pain is intermittent, worse after skipping rope with friends and typically solves after rest or Ibuprofen. She does not recall any inciting injury to the leg. Temp is 37.5C (99.5F). BMI is at the 97th percentile for age. The left hip demonstrates decreased ROM with internal rotation. There is no increased warmth over the joint. The left foot points laterally during ambulation. Which of the following is the most likely underlying cause of this patient's symptoms? A. Bacterial infection of the joint space B. Expansion of malignant cells in bone marrow C. Shearing at the proximal femoral physis D. Transient intraarticular inflammation E. Vasooclusion from intravascular red cell sickling

A. Apnea This neonate has cough, rhinorrhea & lung findings consistent with bronchiolitis, a common respiratory infection in children < 2 that affects the bronchioles. RSV is the most common cause of bronchiolitis & occurs most often in the late fall & winter. Infants & young children tend to also develop inflammation of the lower respiratory tract, resulting in wheezing/crackles & respiratory distress (tachypnea, hypoxia). Fever may be present early in the illness, respiratory symptoms usually peak around days 3-5. Diagnosis is clinical & treatment is supportive (hydration, nasal saline & suction, supplemental oxygen). Most children fully recover within 1-2 weeks. Complications include apnea & respiratory failure, particularly in infants age <2 months.

13-day-old, full-term girl is brought to the office during winter due to a runny nose, cough & fussiness over the past 2 days. Her birth Hx is unremarkable & she has been feeding & growing well. The patient has a sibling in day care, where other children have been ill. Temp is 37.8C (100F) & RR are 70/min. Pulse ox is 95% on room air. Exam shows a crying infant with clear rhinorrhea. Auscultations reveal bilaterally wheezes & crackles. The patient is at greatest risk of developing which of the following complications? A. Apnea B. Bacteremia C. Encephalitis D. Myocarditis E. Seizures

E. Nutritional rickets Rickets presents with crainotabes (soft skull bones), enlarged costochondral joints & progressive genuflects varum. Risk factors for nutritional rivets due to vitamin D deficiency include increased skin pigmentation, limited sun exposure & insufficient dietary intake (exclusive breastfeeding without vitamin D supplementation).

13-month-old boy is brought to the clinic for a well-child visit. The patient has been walking since age 10 months. Solid food intake consists of homegrown baby food, including small portions of fruits, vegetables & grains once a day. He breastfeeds 8-10 times a day. The patient was born full-term & has no chronic medical conditions. He takes no meds, vitamins or supplements. On exam, the anterior fontanel is wide open. The skull bones are pliable to palpation. Bony prominences are palpated at the costochondral junctions bilaterally. Bilateral legs appear bowed & left leg is more bowed than the right. The remainder of his exam is normal. Which of the following is the most likely cause of this patient's findings? A. Achondroplasia B. Congenital syphilis C. Hypothyroidism D. Normal variant E. Nutritional rickets

D. keratitis bacterial conjunctivitis typically presents with erythema & copious yellow or white purulent discharge in a single eye, but can involve both eyes. If left untreated a rare but vision-threatening complication of bacterial conjunctivitis is infectious keratitis or inflammation of the cornea

13-year old boy brought to the clinic by his parents due to copious, purulent discharge from his right eye. The patient's eye was crusted shut this morning. His father was able to open the eye by wiping it with a damp washcloth. The patient has had no fever, ocular pain, visual changes or sick contacts. He has mild myopic and wears contact lenses. VS are normal. Exam shows conjunctival erythema & yellow exudate in the right eye. The conjunctiva of the left eye is also erythematous, but there are no secretions. Visual acuity testing with contacts removed shows 20/80 vision in both eyes, which is unchanged from the last well-child visit at age 12. If left untreated, this patient is at greatest risk for what? A. cavernous sinus thrombosis B. endopthalmitis C. hordeolum D. Keratitis E. uveitis

A. Oshgood-Schlatter disease common cause of knee pain in young adolescents, is caused by traction apophysitis of the tibial tubercle. Patients typically have pain exacerbated by activity & exam reveals prominence & tenderness over the tibial tubercle.

13-year-old boy has had progressive pain of the left knee for 3 months, triggered by playing basketball & relieved by rest. Patient has had no fever, weight loss, or night sweats. VS are normal. PE reveals tenderness over left tibial tubercle, which appears more prominent than the right. The left knee has full ROM & no palpable effusion. Extension of the left knee against resistance & squatting reproduces pain. What is most likely cause of patient's pain? A. Osgood-Schlatter disease B. Patellar tendinitis C. patellofemoral pain D. prepatellar bursitis E. tibial osteomyelitis

C. Physis stabilization with screw fixation This patient has slipped capital femoral epiphysis (SCFE), which is characterized by displacement of the proximal femur relative to the femoral head due to weakening of the femoral growth plate (physis). It is most commonly seen in obese, adolescent (age 10-14) boys, but can occur in girls. SCFE typically presents with insidious onset of hip, thigh or knee pain that leads to limping. Minor trauma, as in this patient, can sometimes exacerbate the pain & prompt the patient to seek medical attention. Exam shows limited internal rotation of the hip and thigh externally rotates during passive hip flexion. A posteriorly displaced femoral head on frog-leg lateral hip XR is diagnostic. Patients should be prescribed non-weight-bearing status & may require bedrest if SCFE is bilateral.

13-year-old girl is brought to the clinic for eval of a limp. She first noticed mild pain in the right groin 2 weeks ago & took ibuprofen, which initially helped. The pain acutely worsened 2 days ago after the patient fell. Temp is 37.2C (99F). When the right hip is flexed, there is external rotation of the hip. When the left hip is flexed, the hip remains in a neutral position. There is no redness or edema overlying the hips. The patient has a limp & bears little weight on her right side when walking. Bilateral hip XR with frog-leg lateral view shows a posteriorly displaced femoral head of the right hip. Which of the following is the best next step in management of this patient? A. Closed reduction of the hip joint B. Oral Methotrexate therapy C. Physis stabilization with screw fixation D. Rest & analgesics as needed E. Total joint replacement

E. XR of the spine Adolescent idiopathic scoliosis is defined as lateral curvature of the spine without a known etiology in a child age > 10. Forward bend test reveals an asymmetric thoracic or lumbar prominence. The 1st step in eval is XR of the spine to determine the degree of curvature & assess skeletal maturity.

13-year-old girl presents to the office for a preparticipation sports physical. Her coach is concerned that the girl's posture seemed abnormal. She has no medical conditions and has not undergone menarche. GMI is at the 50th percentile. PE shows a right-sided lumbar prominence during forward bend test & there is no tenderness of the spine. The remainder of the exam is normal. Which of the following is the best next step in the management of this patient? A. MRI of the spine B. Reassurance & follow-up C. Surgical fixation D. Thoracolumbosacral spinal brace E. XR of the spine

C. order TSH patients with type 1 DM are at increased risk for additional autoimmune disorders, especially autoimmune thyroid disease & celiac disease. Patients should be screened for autoimmune thyroid disease with a serum TSH level for celiac disease with assay for tissue transglutaminase (IgA) antibodies

14-year-old boy is brought in to follow up newly diagnosed type 1 DM. He was diagnosed a week ago when he was admitted to the hospital with DKA. He was discharged on a treatment regimen including insulin glargine & insulin lisper. Since returning home, his finger stick blood glucose levels have ranged from 140-180mg/dL. What should be advised for this patient at this visit? A. initiate insulin pump therapy B. order CBC C. order TSH D. order urine albumin-creatinine ration E. refer for dilated eye exam

E. Recent viral infection This patient with acute, left-sided abdominal pain has anemia, free fluid in the abdomen & signs of shock. These findings are concerning for intraperitoneal hemorrhage due to a ruptured spleen, a potentially fatal condition. Splenic rupture is most commonly caused by high-force, blunt abdominal trauma (MVA) but can also occur spontaneously. The greatest risk factor for atraumatic splenic rupture is stretching of the splenic capsule due to infection or malignancy. Infectious mononucleosis, caused by Epstein-Barr virus, is associated with splenomegaly in up to 50% of cases & splenic rupture within 1 month of symptom onset is rare but a serious complication.

14-year-old boy is brought to the ED by ambulance from his school. The patient is accompanied by his gym teacher, who says the boy suddenly developed left-sided abdominal pain during track practice. Temp is 36.7 (98F), BP is 65/30, HR is 160/min and RR are 30/min. On exam, the patient is pale & diaphoretic. The abdomen is distended & tender to palpation over the LUQ. Bedside US exam reveals free fluid in the abdomen. Hemoglobin is 9g/dL (normal: 13.5-17.5). Further eval of this patient would most likely reveal which of the following? A. Abnormal hemoglobin electrophoresis B. Familial hypertriglyceridemia C. PUD D. Recent ABX use E. Recent viral infection

C. Depression Screening for depression is recommended in all adolescents age 12-18. The Patient Health Questionnaire-2 (PHQ-2) is a 2-item depression-screening scale that is effective for screening adolescents for depression in primary care.

14-year-old girl is brought to the office for a routine health exam prior to starting high school. The patient has had no chronic medical conditions or previous surgeries. menarche was at age 12; the patient is on OCPs for menstrual regulation but is not sexually active. Immunizations are up to date. VS are normal. BMI is 22 kg/m2. PE is unremarkable. This patient should be screened for which of the following? A. Cervical cancer B. Chlamydia infection C. Depression D. Dyslipidemia E. Lead poisoning

C. IV benzo Benzo therapy is the 1st-line treatment to stop status epilepticus and should be administered for seizures lasting > 5 mins. The patients non-compliance is the most likely trigger for her seizure. Historically, status epilepticus has been defined as a single continuous seizure for > 30 mins or multiple seizures occurring in a short period with no return to baseline mental status

14-year-old girl with a history of juvenile myoclonic epilepsy is brought to the ED after 2 generalized tonic-clonic seizures that started approximately 1 hour ago. She has not returned to baseline between seizures. The patient was prescribed Levetiracetam, but her parents suspected that she has no taken it for the past week. On arrival, her airway is patent and she has good respiratory effort. BP is 132/82, HR is 130/min, RR are 16/min and pulse ox is 99% on 2L oxygen via nasal cannula. The patient's eyes are open but she is not responding to commands. Neuro exam shows pupils that are equal but minimally responsive to light. Rhythmic non-suppressible twitching of the mouth, both arms and both legs has persisted for 10 mins. Fingerstick glucose is 115 mg/dL. Which of the following is the best next step in management of this patient? A. Electroencepalogram B. IV Barbiturate C. IV Benzo D. IV Naloxone E. MRI of the head

B. patients age < 18 may be treated for STIs without parental consent. Confidentiality should be maintained & details of the visit cannot be shared with others without the patient's permission This patient has acute cervicitis from a STI (Gonorrhoea or Chlamydia)

15-year-old girl comes to the office due to yellow vaginal discharge for the past 2 days. She has no fever/chills or pelvic pain. She is sexually active. The patient's parents are at work & she comes to the appointment alone. Pelvic exam reveals mucopurulent discharge at the cervical os; there is no cervical motion or adnexal tenderness. UA pregnancy test is negative. The patient is prescribed the appropriate treatment & leaves office. Several hours later, the patient's mother comes to the office asking for an explanation for the visit & for the ABX prescription. What is the most appropriate response? A. "can you explain to me what you think is going on with your daughter?" B. I need your daughter's permission before I discuss her care with you" C. "I recommend you discuss this diagnosis with your daughter" D. "lets schedule another appointment with your daughter to discuss the results together" E. your daughter has a cervical infection & I prescribed ABX"

A. Acute lymphoblastic leukemia ALL is marked by a high concentration of circulating lymphoblasts that are terminal deoxynucleotidyl transferase positive. T-cell ALL usually arises in teens or individuals in their 20s & often presents with a large mediastinal mass.

15-year-old preciously healthy patient arrives at the ED due to progressive SOB. CXR shows a large mediastinal mass & bilateral pleural effusions. Leukocyte count is elevated & peripheral blood smear reveals blast cells. These cells are positive for terminal deoxynucleotidyl transferase. Which of the following is the most likely diagnosis? A. Acute lymphoblastic leukemia B. Acute myeloid leukemia C. Burkitt lymphoma D. Hodgkin lymphoma E. Mantle cell lymphoma

E. Iron deficiency anemia Iron deficiency, commonly caused by excessive milk intake in young children, results in microcytic anemia with an elevated RBC distribution width.

16-month-old full-term girl is brought in to establish well-child care. The patient's family recently moved to the area & have no concerns about her development. The patient weaned from breastfeeding at age 12 months & her current diet consists mainly of whole cow's milk. PE reveals pallor of the conjunctiva but is otherwise unremarkable. Lab results: Hemoglobin: 9.2g/dL (normal: 12-16g/dL) RBC distribution width: 16.5% (normal: 13.5-15.2) MCV: 64 um3 (normal: 80-100um3) Lead: <5 ug/dL Which of the following is the most likely cause of this patient's anemia? A. Anemia of chronic disease B. Beta-thalassemia C. Cobalamin (vitamin B12) deficiency D. Folate deficiency E. Iron deficiency anemia

c. oral amoxicillin-clavulanate Uncomplicated acute otitis media (AOM) is treated with high-dose oral amoxicillin. Patient with recurrent AOM (<30 days after prior infection) are at risk for infection with pathogens that have beta-lactamase resistance & require treatment with either Ceftriaxone (<15 days) or Amoxicillin-clavulanic acid (> 15 days)

16-month-old girl is brought to the office for right ear pain. About 4 weeks ago, the patient had fever & right ear pain & was prescribed a 10-day course of high-dose Amoxicillin for acute otitis media; the course was completed with overall improvement of symptoms & a normal follow-up 2 weeks ago. For the past 3 days, the patient has been tugging at her right ear & crying. She has no nasal congestion, neck stiffness, cough, emesis or other symptoms. She is otherwise healthy & meeting developmental milestones & immunizations are up to date. Temp is 38.2 (100.8F). The patient is alert but appears uncomfortable. There is no rhinorrhea or sinus tenderness. The external auditory canals appear normal, are free from drainage, and are not painful to manipulation. The right tympanic membrane is bulging and erythematous. The left tympanic membrane appears normal. Which of the following is the best next step in management of this patient? A. Myringotomy with tympanovstomy tube placement B. Oral Amoxicillin C. Oral Amoxicillin-Clavulanic acid D. Tympanocentesis with culture E. Watchful waiting

B. Infertility A varicocele typically presents as a soft, irregular mass that increases in size with standing & Valsalva. Varicoceles are associated with increased risk for infertility; for patients with testicular atrophy or changes in some analysis, surgical venous ligation can improve fertility.

16-year-old boy comes to the office due to occasional discomfort & heaviness of his scrotum over the past several months. He has had no trauma, dysuria or urethral d/c. The patient is not sexually active. Vitals are normal. On PE, an irregular, ropy mass is noted on both sides of the upper scrotum. The mass increases in size with Valsalva & reduces on supine positioning. There is no erythema, ulcers or tenderness of the scrotal skin & no inguinal LAD. The rest of the exam is normal. Surgical correction is discussed with the patient & his parents. The primary reason to perform surgery is to prevent which of the following complications in this patient? A. Chronic epididymitis B. Infertility C. Intestinal strangulation D. Testicular cancer E. Torsion of the testis

D. Lack of right testicular retraction on stroking of the medial thigh Testicular torsion is a surgical emergency that presents with acute-onset scrotal pain & swelling, classically with an absent cremasteric reflex.

16-year-old boy is brought to the ED due to severe scrotal pain. The patient had an abrupt onset of scrotal pain 6 hours ago that has continued to worsen. The pain is now severe, making walking difficult. The patient is nauseated & vomited once but has had no abdominal or flank pain. Temp is 37.2C (99F). The abdomen is soft, no tender & has no rebound or organometallic. Bowel sounds are active. Genital exam reveals an exquisitely tender & swollen right testicle. Elevation of the testis exacerbates the pain. What is the most likely additional finding in this patient? A. Area of trans illumination superior to the right testis B. Cystic mass with calcification on testicular US C. Increased blood flow to the epididymis on scrotal US D. Lack of right testicular retraction on stroking of the medial thigh E. Pain the right lower quadrant with passive ipsilateral hip extension

B. Hodgkin lymphoma Prolonged fever & LAD (particularly supraclavicular) are common manifestations of Hodgkin lymphoma. Reed-Sternberg cells on lymph node biopsy are diagnostic.

16-year-old boy is brought to the clinic for eval of intermittent fevers. He first noticed symptoms after returning from Pennsylvania 1 month ago. Initially, the fevers occurred every few days; however, for the past 2 weeks, he has had a daily fever. He has no dyspnea, night sweats or weight loss. He is sexually active & has had 3 lifetime partners. Temp is 38.3C (100.9F). Multiple small right anterior cervical lymph nodes are palpable. The right supraclavicular fossa has a solitary, firm, 2.5 cm (1-in) palpable lymph node. Biopsy of the lymph node shows large lymphocytes with bilobed nuclei (Reed-Sternberg cells). Which of the following is the most likely diagnosis for this patient? A. Acute HIV infection B. Hodgkin lymphoma C. Lyme disease D. Sarcoidosis E. Tuberculosis

A. Borrelia burgdorferi infection Lyme arthritis is the hallmark of late Lyme disease due to Borrelia burgdorferi infection. The presentation is most commonly a monoarticular arthritis of the knee that occurs in a weight-bearing, afebrile patient. Synovial fluid analysis shows inflammation but Gram stain & culture are usually negative

16-year-old comes to the office in October due to right knee swelling that began yesterday following soccer practice. He has mild associated stiffness. Over the last several months, he has had fatigue with occasional flulike illnesses & pain occurring in variable joints. The patient spent the summer in Maine. He is sexually active & uses condoms. VS is normal. The patient can bear weight. The right knee is tender, warm & has a large effusion. All other joints are normal. Joint aspiration fluid has a leukocyte count of 20,000/mm3 (50% neutrophils) & no organisms on Gram stain. What is most likely cause of patient's knee swelling? A. Borrelia burgdorferi infection B. disseminated gonococcal infection C. prior Chlamydia trachoma's infection D. prior streptococcal throat infection E. Staph joint infection

E. Video electroencephalography Patients with psychogenic nonepileptic seizures have behaviors that are unusual for true seizures, such as rapid head turning & synchronous thrashing without loss of consciousness or cyanosis. Video electroencephalography is typically diagnostic.

16-year-old girl is seen in the clinic for eval of seizures. The patient was diagnosed with a seizure disorder at age 5, and her seizures have been well controlled until recently. For the last month, she has been having more frequent seizures, now up to 4 episodes a day. When the seizures occur, the patient suddenly falls to the ground & moves her head rapidly from side to side. She has no associated injuries or personal cyanosis during the episode. Which of the following would most likely diagnose this patient's condition? A. Lumbar puncture B. MRI of the brain C. Serum lead level D. Urine toxicology screen E. Video electroencephalography

D. Orchitis Mumps presents with fever & parotitis. Although mumps is generally self-limited, complications can include orchitis, aseptic meningitis, pancreatitis and deafness. Complications are more common in older or unvaccinated individuals.

17-year-old boy is brought to the clinic due to fever & facial pain. 2 days ago, symptoms of fever, myalgia & fatigue began. This morning, pain & fullness developed in the right cheek. Immunizations are not up to date. The patient is not sexually active. Temp is 38.3C (100.9F). Exam reveals an alert, uncomfortable-appearing adolescent. Tenderness & fullness are present in the right cheek, which obscures the angle of the mandible. Bilateral tympanic membranes are clear. The oropharynx has no erythema, exudate or tonsillar enlargement. No cervical lymph nodes are palpable. This patient is at greatest risk for developing which of the following complications? A. Glomerulonephritis B. Lymphopenia C. Myositis D. Orchitis E. Splenic rupture

B. Acute iron poisoning Acute iron poisoning initially presents with symptoms of direct injury to the GI tract (hematemesis) & is followed by shock & anion gap metabolic acidosis. Hepatic necrosis & bowel scarring/obstruction may also occur. Clinical manifestation- - abdominal pain, hematemesis, diarrhea - shock - liver necrosis Diagnostic findings- - anion gap metabolic acidosis - elevated serum iron - radiopaque pills on abdominal XR Treatment- - Deferoxamine - whole bowel irrigation

17-year-old girl is brought to the ED by her older sister who found the patient writhing on the floor with severe abdominal pain. Since then, the patient has had several episodes of coffee-ground emesis & 2 episodes of dark green diarrhea. She has major depressive disorder & was hospitalized 1 month ago for a suicide attempt. The patient also has anemia, treated with supplementation. She is sexually active & her LMP was 3 weeks ago. Temp is 37C (98.6F), BP is 80/50m HR is 120/min & RR are 28/min. On PE, she is lethargic & appears uncomfortable. Capillary refill is 5 seconds. The abdomen is soft but diffusely tender to palpation. Which of the following is the most likely diagnosis? A. Acute appendicitis B. Acute iron poisoning C. Irritable bowel syndrome D. Ruptured ectopic pregnancy E. Tricyclic antidepressant OD

C. Nontypeable H. Influenzae Common pathogens responsible for AOM include strep pneumoniae, nontypeable H. Influenzae & Moraxella catarrhalis. H. Influenzae commonly causes refractory AOM & otitis-conjunctivitis syndrome (concurrent purulent conjunctivitis & AOM)

18-month-old boy is brought in for right ear pain. For the past 3 days, the patient has been tugging on his right ear & crying. He has no nasal congestion, neck stiffness, cough, emesis or other symptoms. He is otherwise healthy & immunizations are up to date. Temp is 38.5C (101.3F). The conjunctiva are injected bilaterally & have purulent discharge. There is no rhinorrhea or sinus tenderness. The external auditory canal appear normal, are free from drainage & are not painful to manipulation. The right TM is erythematous, bulging & immobile on pneumatic air insufflation. The left TM appears normal. Which of the following is the most likely cause of this patient's current symptoms? A. Adenovirus B. Moraxella catarrhalis C. Nontypeable H. Influenzae D. Pseudomonas aeruginosa E. Staph aureus

B. Air contrast enema Intussusception is a pediatric emergency in which a segment of bowel telescopes into an adjacent segment, causing characteristic episodic pain, emesis, "current jelly" stool & lethargy. Rapid diagnosis & treatment with US-guided air contrast enema is critical for avoid ischemia & peritonitis

18-month-old boy is brought to the ED b/c he has become difficult to arouse for about an hour. For the past 6 hours, the boy has had 10-15 minute episodes of crying inconsolably that resolve & the patient is asymptomatic. His appetite is decreased 7 he has had multiple episodes of emesis. His only bowel mov't today was a dark red & "sticky" stool that contained streaks of blood. Temp is 36.7 (98F); BP: 100/62; HR: 128/min; RR: 18/min. On PE he is lethargic. The abdomen is soft & nondistended with mild RLQ tenderness with a palpable mass & hypoactive bowel sounds. Which is the best next step in management of this patient? A. Abdominal CT scan with contrast B. Air contrast enema C. Anorectal manometry D. Barium enema E. Merkel scan

D. Parainfluenza virus Croup is a viral illness characterized by fever; rhinorrhea; sudden onset of a harsh, barking cough; hoarseness; and stridor. The most common etiology is parainfluenza virus.

18-month-old boy is brought to the clinic for eval of cough. He has had clear rhinorrhea since yesterday, fever & a harsh cough began suddenly this morning, but no episodes of associated vomiting. He is up to date on his vaccinations. Temp is 38.3C (100.9F), HR is 134/min, RR are 34/min. PE shows an alert child with audible stridor who has multiple episodes of loud, forceful coughing. The nares are filled with white nasal d/c & postnasal drip is present in the posterior oropharynx. There are no rales or wheezes in the lungs. Intercostal & suprasternal retractions are present. Which of the following organisms is the cause of this patient's symptoms? A. Bordetella pertussis B. Haemophilus influenzae type B C. Influenza virus D. Parainfluenza virus E. Streptococcus pneumoniae

B. edema & narrowing of the proximal trachea croup is a viral infection in which edema & narrowing of the proximal trachea result in a barky cough & inspiratory stridor. Treatment with CCS (with or without nebulized epinephrine, depending on severity) decreases airway inflammation.

18-month-old girl is brought to the office due to difficulty breathing. Patient has had a runny nose & congestion since yesterday. She started coughing last night & it has worsened throughout the day. The patient is drinking fluids & voiding normally. Vaccinations are up to date. Temp is 37.8C (100F), BP: 90/60, RR: 30/min. Pulse ox is 97% on room air. On exam, the patient's cough is harsh & inspiratory stridor is heard when she is crying but resolves at rest. The lungs are CTA. Which is the most likely cause of patient's symptoms A. bronchoconstriction & small airway inflammation B. edema & narrowing of proximal trachea C. edema of epiglottis & aryepiglottic folds D. foreign body obstruction within bronchus E. infectious pulmonary lobar infiltrate

B. Hirschsprung disease Hirschsprung disease should be suspected in any newborn with failure to pass meconium within 48 hours after birth, especially in the setting of Down Syndrome. The presentation consistent with complete bowel obstruction (poor feeding, abdominal dissension, absent air in the rectum)

2-day-old boy in the newborn nursery was born at 39 weeks gestation to a 38-year-old woman by vaginal delivery. Since birth, the boy has shown little interest in feeding. He spit up the last feed but has not vomited. He has not yet passed meconium. PE shows a newborn with generalized hypotonia, a flat facial profile, low-set folded earlobes & a single transverse crease on each palm. His abdomen is firm & distended. DRE is notable for a tight anal canal & relieves an expulsion of gas & stool. Abdominal XR shows markedly distended loops of bowel with no gas in the rectum. Which of the following is the most likely cause for this infant's findings? A. Congenital hypothyroidism B. Hirschsprung disease C. Intussusception D. Maternal epidural analgesia E. Meconium ileus

E. tunica vaginalis A communicating hydrocele results when serous peritoneal fluid accumulates within the tunica vaginalis in the setting of a patent processes vaginalis. It presents as a painless swelling of the scrotum that transilluminates on exam.

2-week-old boy is brought in for his first newborn eval. He has had persistent swelling of the left scrotum but with no noticeable discomfort or irritability. He is voiding & stooling normally. PE shows bilateral descended testicles with an enlarged & fluctuant left hemiscrotum that transilluminates brightly. No inguinal or abdominal masses are present. The most likely cause of this newborn's condition is a swelling in which of the following locations? A. caput of the epidymis B. external spermatic fascia C. pampiniform plexus D. tunica albuginea E. tunica vaginalis

B. Consult orthopedic surgeon patients with developmental dysplasia of the hip should be referred to an orthopedic surgeon for treatment with a Pavlik harness, which promotes normal acetabular development

2-week-old girl is brought to the office for a routine health visit. The patient was born to a 29-year-old primigravida at 40 weeks gestation by cesarean delivery for breech presentation. Birth weight was large for gestational age at 4kg (8lb 13oz); current weight is 4.2kg (9lb 4oz). A complete PE reveals a palpable "clunk" of the right hip during attempted dislocation. An US of both hips is ordered & reveals a dislocated right hip with a flat acetabulum. Which is the best next step in management of this patient? A. Consult genetic specialist B. Consult orthopedic surgeon C. Obtain right hip radiograph D. Obtain spinal canal US E. Reassure & observe only

E. Topical nystatin drops Oropharyngeal candidiasis (thrush) is common in infants & typically presents with white plaques on the tongue, palate &/or buffalo mucosa with underlying inflamed mucosa. 1st-line treatment is topical nystatin

2-week-old, full-term is brought in for veal of mouth lesions that were noticed yesterday. The infant has been breastfeeding for 10 mins per side every 2 hours but is less interested in feeding today. The mother had normal prenatal lab testing & takes no meds. Exam shows white plaques on the buffalo mucosa & tongue that scrape off with a tongue depressor, revealing erythematous underlying mucosa. No other abnormalities are seen. Which of the following is most appropriate management of this infant? A. HIV testing B. Lesion swabbing & culture C. Oral systemic fluconazole D. Reassurance & observation E. Topical nystatin drops

E. Normal development 2-year-old children with appropriate language development should have a vocab of > 50 words, combine words into 2-word phrases & follow 2-step commands. In addition, a stranger should be able to understand at least half of a 2-year-old's speech

2-year-old boy is brought in b/c his parents are concerned about his development. The boys says about 70 words with some 2-word phrases. The patient likes making beeping sounds when playing with toy trucks, but he does not play with his sister, only alongside her. He points to a toy across the room, looks at his mother & smiles, then brings the toy over to show her. When the health care provider approaches, he runs to hide behind his father & initially refuses to cooperate or make eye contact. When his mother tries to help him onto the exam table, he pushes her away & says, "me do." Which of the following is the most likely diagnosis in this patient? A. Autism spectrum disorder B. Cognitive delay C. Conductive hearing loss D. Language delay E. Normal development

D. Mastoid air cells Acute mastoiditis, a bacterial infection of the mastoid air cells, is a complication of AOM. Patients typically have fever, protrusion of the auricle, post-auricular erythema, swelling & tenderness.

2-year-old boy is brought in by his parents due to worsening ear pain. The boy started tugging on his ear 3 days ago & has had a fever. This morning, the patient had increased pain. He has been less active than normal but has had no vomiting, diarrhea or rash. Immunizations are up to date & he takes no daily meds. Temp is 40C (104F). The patient is lethargic & irritable. PE shows swelling, erythema & tenderness to palpation posterior to the right ear. Otoscopy reveals an erythematous & bulging TM with loss of normal landmarks & light reflex. Which of the following structure involvements is most likely responsible for this patient's presentation? A. Epidural space B. External auditory canal C. Labyrinth D. Mastoid air cells E. Meninges

A. erythema infectiosum a self-limiting infection caused by parvovirus B19. Presentation begins with fever & nonspecific prodrome (cough, congestion, malaise) followed by a bright red facial rash ("slapped cheek") & a reticulated rash on the trunk & extremities

2-year-old boy is brought in to the office by his father for eval of a rash. His father 1st noticed a red rash on the boy's stomach this morning & it has now spread to the arms. Last week, the patient has rhinorrhea, cough & a fever of 38.3C (101F) for 2 days. Yesterday, the face appeared "flushed" but his temp was normal & the redness resolved without intervention. The patient attends daycare during the week. Temp: 36.7C (98F). Mucous membranes are moist, & no oropharyngeal lesions are noted. The neck is supple & without LAD. Skin exam reveals an erythematous, non blanching, reticulated rash on the upper & lower extremities. No joint swelling is noted, & the extremities show full ROM. What is the most likely diagnosis? A. erythema infectiosum B. Kawasaki disease C. measles D. scarlet fever E. systemic juvenile idiopathic arthritis

D. Nebulized racemic Epinephrine Croup, a viral infection that causes subglottic edema & narrowing, presents with a barky cough & inspiratory strider. Patients with stridor at rest are treated with CCS & epinephrine

2-year-old boy is brought to the ED due to difficulty breathing that started 6 hours earlier. The patient has had rhinorrhea, nasal congestion & a dry cough that sounds "like a barking dog" for the past day. Temp: 37.7C (99.9F); RR: 24/min. Pulse ox on room air is 99%. The patient has inspirations stridor when crying. He is diagnosed with croup, administered oral Dexamethasone & observed in the ED. On reassessment 30 mins later, oxygen saturation is 96% on room air & RR: 32/min. The patient has inspiratory strider at rest & subcostal retractions. Which is the best next step in management of this patient? A. CXR B. Intubation & mechanical ventilation C. Nebulized Albuterol D. Nebulized racemic epinephrine E. Supplemental oxygen via face mask

E. Watchful waiting Serous otitis media is a persistent, serous, middle ear fluid collection, often seen after appropriate treatment of an acute episode of otitis media. Initial management is watchful waiting b/c the effusion generally resolves spontaneously within 3 months.

2-year-old boy is brought to the office for follow-up. He was last seen 3 weeks ago for an episode of acute otitis media that was treated with a 10-day course of high-dose Amoxicillin. Exam at that time showed copious clear rhinorrhea & a bulging, erythematous right tympanic membrane. His mother reports that his fever has resolved, he is no longer cranky or indicates ear pain & his appetite & activity level have returned to baseline. Temp is 36.7C (98.1F), HR is 106/min and RR are 18/min. The left tympanic membrane is normal; the right tympanic membrane is slightly retracted with yellow fluid behind the membrane & decreased mobility on pneumatic otoscopy. Which of the following is the best next step in management of this patient? A. Amoxicillin-Clavulanic acid B. Audiometry C. Combo antihistamine & nasal decongestant D. Myringotomy & insertion of tympanovstomy tube E. Watchful waiting

E. Tetralogy of Fallot TOF presents with hypercyanotic "tet" spells due to obstruction of the right ventricular outflow tract during exertion and a harsh crescendo-decrescendo systolic murmur over the left upper sternal border.

3-month-old boy is rushed to the ED for eval of cyanosis. The baby appeared "bluish-pale" during a bottle feeding this morning & still appears "dusky," The parents say that this color change has occurred during crying or feeding for the past few days. The patient also seems to tire & breathe faster after few minutes of feeding. Temp is 36.7C (98F), BP is normal and equal in all 4 extremities, pulse is normal and respiratory rate is slightly elevated. Pulse oximetry show an oxygen sat of 80% on room air. The infant is crying and the cyanosis is most prominent in the lips and tongue. Auscultation reveals a single S2, and a harsh systolic ejection murmur is heard at the left upper sternal border. No digital clubbing is present. Cap refill is < 2 secs. Which of the following is the most likely diagnosis in this patient? A. Atrial septal defect B. Coarctation of the aorta C. Hypoplastic left heart syndrome D. Isolated ventricular septal defect E. Tetralogy of Fallot

E. Tetralogy of Fallot TOF presents with hypercyanotic "tet" spells due to obstruction of the right ventricular outflow tract during exertion and a harsh crescendo-decrescendo systolic murmur over the left upper sternal border. "Tet" spells are episodes of hypoxemia and cyanosis and commonly occur during exertion, feeding and agitation (crying) which can incase pulmonary vascular resistance, resulting in complete right ventricular outflow tract obstruction (due to pulmonary valve stenosis).

3-month-old boy is rushed to the ED for eval of cyanosis. The baby appeared "bluish-pale" during a bottle feeding this morning & still appears "dusky." The parents say that this color change occurred during crying or feeding for the past few days. The patient also seems to tire & breathe faster after a few minutes of feeding. Temp is 36.7C (98F), BP is normal and equal in all 4 extremities, pulse is normal and respiratory rate is slightly elevated. Pulse ox shows an oxygen saturation of 80% on room air. The infant is crying and the cyanosis is most prominent in the lips and tongue. Auscultation reveals a single S2 and a harsh systolic ejection murmur is heard at the left upper sternal border. No digital clubbing is present. Cap refill is < 2 secs. Which of the following is the most likely diagnosis in this patient? A. Atrial septal defect B. Coarctation of the aorta C. Hypoplastic left heart syndrome D. Isolated ventricular septal defect E. Tetralogy of Fallot

B. Azithromycin This patient has a severe cough with apnea & posttussive emesis that is concerning for pertussis, a highly contagious infection that is caused by Bordetella pertussis and transmitted through respiratory droplets. Infants who have not received their acellular pertussis vaccine are at high risk for infection. It typically progresses from a mild cough & rhinorrhea to a harsh, whooping cough with posttusive emesis before gradual symptom resolution over weeks to months. In infants, the initial phase may be short/mild or absent, whereas the paroxysmal phase is often particularly severe with gagging, gasping, cyanosis & life-threatening apnea. Fever is uncommon. Infant with suspected pertussis should receive immediate empiric treatment with a macrolide ABX (Azithromycin) while awaiting confirmatory testing.

3-week-old girl is brought to the ED due to apnea & cyanosis. Her mother reports multiple episodes over the last 48 hours of gagging & gasping for air after severe coughing. During the last event, the patient started coughing harshly during a feed, vomited & then stopped breathing. She became cyanotic for approx 12 secs, then quickly returned to normal. The patient has a 1-week history of clear rhinorrhea & increasing cough that is followed by emesis. Temp is 37.2C (99F), BP is 85/52, HR is 110/min and RR are 45/min. She is well developed & active. There is nasal congestion & clear rhinorrhea. Lung sounds are clear bilaterally. Cardiac exam is normal. Which of the following is the most appropriate empiric pharmacotherapy for this patient's condition? A. Amoxicillin B. Azithromycin C. Furosemide D. Oseltamivir E. Ranitidine

B. Glomerulonephritis Impetigo presents as erythematous papules & pustules covered with a honey-colored crust, typically on the face or upper extremities. A potential sequel are of group A streptococcal impetigo is acute poststreptococcal glomerulonephritis

3-year-old boy is brought in by his mother due to a rash. 3 days ago, the patient developed a few red papules on his chin; they subsequently evolved into pustules & today developed an overlying crust. Several new pustules have appeared at the corner of his nose. He has had no fever. There are several children with a similar rash at his day care facility. The patient has no chronic medical conditions & takes no meds. Vaccinations are up to date. VS are normal. PE reveals 5 erythematous papules covered with a thick, yellow crust on the chin & 2 pustules on the right nasal ala. The remainder of the exam is normal. This patient is at greatest risk of developing which of the following complications? A. Encephalitis B. Glomerulonephritis C. Myocarditis D. Thyroiditis E. Uveitis

C. Blood lead level Lead poisoning in children can cause behavioral problems (anxiety, attention-deficit hyperactivity, loss of milestones), cramps abdominal pain, vomiting & constipation. Diagnosis is with a venous blood lead level. Treatment of moderate to severe lead poisoning includes chelation

3-year-old boy is brought in for eval of his behavior. Over the past 3 months, he has become fussier & moodier than usual. He was previously very talkative but his mother has noticed a decline in his vocab. She cannot identify any social stressors or changes at home except that the family moved into a different apartment building approx 5 months earlier. The mother is unsure of the year it was built. The child has also been very constipated & had intermittent abdominal pain. Yesterday he had 2 episodes of non-bloody nonbilious emesis & has been eating less than usual. Which of the following is the best next step in management of this patient? A. Abdominal XR B. Attention deficit hyperactivity disorder assessment C. Blood lead level D. Child protective services consult E. Hearing screen

E. Wilms tumor Wilms tumor is the most common renal malignancy in children. Although it usually presents as an asymptomatic abdominal mass, hematuria may occur in up to 1/4th of patients. Abdominal pain & HTN may also present. Although the lungs are the most common site of metastatic spread, children rarely have pulmonary symptoms

3-year-old boy is brought in for eval of red-tinged urine, which his mother noticed earlier this week. The patient has had multiple episode since but has no associated dysuria. He has had no prior injury but was recently treated with ABX for streptococcal pharyngitis. The patient has had no chronic medical conditions. Temp is 36.7C (98.1F). Exam shows a well-appearing & well-nourished boy. There is a firm, nontender, palpable mass in the left abdomen. UA reveals: Blood: 2+ Glucose: negative Protein: negative Bacteria: negative RBC: 30/hpf WBC: 1/hpf Which of the following is the most likely diagnosis of this patient? A. IgA nephropathy B. Neuroblastoma C. Postreptococcal glomerulonephritis D. Renal cell carcinoma E. Wilms tumor (nephroblastoma)

E. Upright positioning after feeds GERD is a common physiologic phenomenon in infants & is characterized by split-up with appropriate weight gain & no pain with feeds. 1st line management includes reassurance & lifestyle modifications (upright positioning after feeds).

4-month-old boy is brought to the office due to concerns about feeding. The patient has been drinking formula since birth and drinks 3-4oz every 2-3 hours. Within 15 mins of each feed, the boy vomits and dribbles a yellowish liquid that looks like partially digested formula. The patient has 2 soft, yellow stools each day. He has no coughing or choking with feeds. Review of his growth chart shows a birth weight at the 41st percentile, with today's weight at the 44th percentile. PE reveals a well-nourished infant. The anterior fontanelle is soft & flat. The abdomen is soft, nontender and nondistended. The mucous membranes are moister & skin turgor is normal. Which of the following is the best next step in management of this patient? A. Abdominal US B. H2-receptor blocker C. Nissen fundoplication D. Upper endoscopy E. Upright positioning after feeds

C. Epiglottis Epiglottis is a rare but potentially fatal infection that presents with acute onset of fever, sore throat, and signs of upper airway obstruction (stridor, drooling). Symptoms often develop over hours without a notable prodrome. Impending signs of respiratory failure include anxiety, worsening stridor and a muffled/hoarse "hot potato" voice. Patients may display tripping positioning (leaning forward, neck hyperextension) to maximize airway diameter. The anterior neck near the hyoid bone may be tender, and oropharyngeal exam is typically normal other than pooled oral secretions. Plain XR may help confirm the diagnosis by revealing an enlarged epiglottis. Diagnosis is confirmed with direct visualization of edematous epiglottis.

3-year-old boy is brought to the ED due to a sore throat. The boy woke up with the sore throat this morning & has refused to eat. Temp is 38.7C (101.7F) and RR are 28/min. The patient is sitting still on his mother's lap and appears scared. He has a hoarse voice, rhinorrhea & mild stridor. Tympanic membranes are clear bilaterally. Exam of the posterior pharynx shows no erythema or tonsillar exudate. The anterior neck is tender to palpation. Lung exam reveals transmitted upper airway sounds that are equal bilaterally without crackles or wheezes. Lateral neck x-ray shows supraglottic swelling and enlarged epiglottis (thumb print sign). E Which of the following is the most likely diagnosis in this patient? A. Bacterial tracheitis B. Croup C. Epiglottis D. Peritonsilar abscess E. Retropharyngeal abscess

B. Coagulation studies This child's history of easy bruising & acute onset of joint effusion following a minor trauma (suggestive of hemarthrosis) raise concern for hemophilia, a coagulation disorder. Inheritance is X-linked recessive and common in young boys. Clinical features: delayed/prolonged bleeding after mild trauma - hemarthrosis, intramuscular hematomas - GI or GU tract bleeding - intracranial hemorrhage Initial screening tests for Hemophilia include: coagulation studies: - prolonged/elevated activated PTT - absent/ decreased factor VIII (hemophilia A) or factor IX (hemophilia B) activity - normal PT & a CBC: - normal platelet count Treatment: factor replacement Desmopressin for mild Hemophilia A

3-year-old boy is brought to the ED due to right knee swelling. The knee swelled immediately after patient fell while playing outside. The patient also developed a large bruise to his left hip last week when he fell from the couch. Temp is 36.7C (98.1F). Exam shows an uncomfortable-appearing boy with significant swelling of the right knee. The patient has pain with knee movement and limited ROM. An area of ecchymosis is seen on the left hip. The remainder of the PE is normal. Hemoglobin: 13 g/dL Platelets: 187,000/mm3 Leukocytes: 8,500/mm3. XR of right nee shows a large effusion but no fracture. Which of the following is the best next step in management of this patient? A. Bone marrow eval B. Coagulation studies C. Complete skeletal survey D. Reassurance & observation E. Synovial fluid Gram stain & culture

C. Prednisone Minimal change disease is the m/c cause of nephrotic syndrome in young children. Renal biopsy is not required for initial diagnosis as the condition is highly responsive to steroids.

3-year-old boy is brought to the office with a 1-week history of generalized edema that is gradually worsening. The patient has not been ill recently, and he has no chronic medical conditions. Temp is 36.7C (98F), BP is 110/80, HR is 85/min and RR are 18/min. On exam, he has periorbital edema and 1+ pretibial pitting. The remainder of the PE is normal. Lab results: Sodium: 140 mEq/L (normal: 136-145) Potassium: 3.7 mEq/L (normal: 3.5-5.0) Creatinine: 0.8 mg/dL (normal: 0.6-1.2) Total Protein: 5.5 g/dL (normal: 6.0-7.8) Albumin: 2.1 g/dL (normal: 3.5-5.5) UA shows 4+ proteinuria and no RBCs. Which of the following is the best next step in the management of this patient? A. Echocardiography B. IV albumin C. Prednisone D. Renal biopsy E. Renal scintigraphy

D. Tetralogy of Fallot Tetralogy of Fallot is the m/c cyanotic congenital heart defect. It is characterized by the following 4 cardiac malformations: 1.) Pulmonary stenosis (leading to right ventricular outflow tract obstruction) 2.) Right ventricular hypertrophy 3.) Overriding aorta 4.) Ventricular septal defect Affected patients are usually diagnosed shortly after birth when classic exam findings such as a harsh systolic ejection murmur & a systolic thrill at the left sternal boarder prompt further investigation with echocardiogram. Infants with significant pulmonary stenosis are often cyanotic at birth due to right-to-left shunting. Less severe pulmonary stenosis results in left-to-right shunting in infancy/childhood & the absence of early cyanosis. 1 of the defining characteristics is that the degree of pulmonary stenosis can fluctuate rapidly. Without surgical repair, children may experience hypercyanotic ("tet") spells that manifest with agitation, tachypnea, syncope, seizures & possibly death. During the onset of these spells, children will often squat in an effort to relieve the cyanosis (improve oxygenation). Squatting increases systemic vascular resistance & cardiac afterload, temporarily reducing right-to-left shunting by forcing more blood through the stenotic pulmonary artery.

3-year-old boy whose family immigrated to the US several months ago is being evaluated for extreme fatigue & occasional syncope. His mother says that he has experienced episodes of agitation & tachypnea since birth & that these episodes have progressively worsened to the point that he sometimes loses consciousness. Cardiac exam shows a harsh systolic ejection murmur heard of the left sternal border. His O2 sat is 84% while supine & 91% while squatting. Which of the following is the most likely diagnosis? A. Aortic coarctation B. Atrial septal defect C. Mitral stenosis D. Tetralogy of Fallot E. Ventricular septal defect

D. Pericardial effusion This patient with recent cardiac surgery developed tachycardia, tachypnea, distant heart sounds and enlargement of the cardiac silhouette on CXR, suggestive of pericardial effusion. A pericardial effusion occurs when inflamed pericardial cells produce increased fluid, which accumulates in the pericardial sac around the heart. This is common a few weeks after cardiac surgery due to immune-mediated inflammatory damage to the pericardium (postpericardiotomy syndrome). Small pericardial effusions are typically asymptomatic bc the pericardium stretches to accommodate fluid. Larger effusions can compress the heart & impair cardiac filling. Infants can develop decreased appetite and vomiting whereas older children may present with pericardial friction rub & pleuritic chest pain. Fever may be present due to inflammation & tachypnea & tachycardia occur in response to decreased cardiac output. The effusion obscures auscultation, causing distant "muffled" heart sounds on exam. Fluid accumulation around the heart causes an enlarged cardiac silhouette on CXR. Echo confirms diagnosis.

4-week-old male infant with hypoplastic left heart syndrome has had difficulty feeding & an episode of vomiting over the last 24 hours. The patient underwent palliative heart surgery at age 1 week; the procedure involved craniotomy & extensive intracardiac & aortic arch repair. His postop course was uncomplicated, and he has remained in the hospital for weaning off oxygen & titration of cardiac meds. Temp is 38.3C (101F), BP is 80/40, HR is 175/min and RR are 70/min. Pulse ox is 80% on room air. Cardiopulmonary exam is notable for distant heart sounds, tachycardia & tachypnea. CXR from 1 week ago is compared to today's CXR. CXR from 1 week ago appears normal. Today's CXR shows shows an enlarged cardiac silhouette and a "stomach bubble." Which of the following is the most likely cause of this patient's acute presentation? A. Congestive heart failure B. Endocarditis C. Mediastinitis D. Pericardial effusion E. Pulmonary embolism

A. High pH, high PaCO2, high HCO3 Pyloric stenosis presents at age 3-5 weeks with no billows, projectile vomiting after each feed. Protracted vomiting produces hypochloremic, hypokalemic metabolic alkalosis

4-week-old, full-term boy is brought to the ED due to vomiting. The vomiting occurs after feeds & its frequency & force have increased the past 6 days. However, the patient continues to bottle-feed every 1-2 hours. He previously had soft, yellow stools daily but has had no stool for the last 2 days. Temp: 37.2C (99F); BP: 70/30; HR: 182/min; RR: 10/min. Exam shows a thing, sleepy infant with a sunken anterior fontanelle & dry mucous membranes. Cardiac exam reveals tachycardia but no murmurs or gallops. The abdomen is no tender & nondistended with a small, firm, palpable abdominal mass. Which of the following lab finding is expected in this patient? A. High pH, high PaCO2, high HCO3 B. High pH, low PaCO2, low HCO3 C. Low pH, high PaCO2, high HCO3 D. Low pH, low PaCO2, low HCO3 E. Normal pH, normal PaCO2, normal HCO3

D. US of abdomen a secondary varicocele should be suspected in a prepubertal boy with a soft, coiled ("bag of worms"), right-sided scrotal mass that fails to decompress when supine. Abdominal US is indicated to evaluate for anatomical causes leading to venous compression (such as venous thrombus or abdominal mass)

4-year-old boy brought to clinic for eval of scrotal swelling. His mother first noticed swelling near right testicle after giving him a bath 2 days ago. Mass is not painful, the patient has had no difficulty urinating. He has had no recent testicular trauma & patient is otherwise healthy & takes no meds. Temp is 37.1C (98.8F). Initial BP: 160/96; a repeat assessment is 152/92. GU exam reveals Tanner stage 1 development with bilateral testicles within the scrotum. There is a palpable, contender, soft, coiled mass superior to the right testicle. The mass does not transilluminate & does not decrease in size when patient is supine. Remainder of PE is normal. UA reveals no blood, nitrites or leukocyte esterase. Which is next best step in management of this patient? A. Bx of scrotal mass B. reassurance & observation only C. surgical orchiectomy D. US of abdomen E. urine culture

D. MRI of the brain Direct spread of bacteria from otitis media or mastoiditis can cause a temporal brain abscess. The presentation can include severe headache, morning vomiting, fever & focal neurological deficits. Diagnosis is confirmed by contrast-enhanced brain CT or MRI.

4-year-old boy is brought in for worsening ear pain that started after completing a course of ABX for otitis media, 2 weeks ago. For the past several nights, he has also been awakened by headaches; however, he feels better after vomiting in the morning. Temp: 38C (100.4F), BP: 100/60, HR: 95/min. PE reveals an irritable child. Fundoscopic exam is normal & extraocular motion is intact. The left TM is bulging, erythematous & opaque. The left mastoid is tender, erythematous & swollen. Which is the next best step in management of this patient? A. ABX & follow-up in 2 days B. Lumbar puncture C. Mastoidectomy D. MRI of the brain E. Skull XR

B. MRI of brain Focal seizures may be secondary to a structural abnormality of the brain & concomitant nocturnal headaches raise suspicion for an underlying brain tumor. MRI of brain is gold standard for eval.

4-year-old boy is brought to the ED after a seizure. The boy was resting on the couch when his left arm started writhing & stiffening. His mother tried to talk to him, but he did not respond during the episode. The seizure lasted approx 10 mins & ended before emergency personnel arrived. The patient has had headaches for the last few weeks, which often awaken him from sleep. Otherwise, he has no chronic medical conditions. The patient's mother takes opioids for chronic back pain. Temp is 37.2C (99F). On exam, the patient appears tired but is cooperative. Neuro exam, including fundoscopy is normal. Which of the following is the best next step in management of this patient? A. Lumbar puncture B. MRI of brain C. Plasma & urine amino acids D. Urine toxicology screen E. Video electroencephalography

E. Patching of the right eye Strabismus, or ocular misalignment, can be detected by asymmetric red and corneal light reflexes, as well as eye deviation on the cover test. Management includes correcting refractive errors and strengthening the affected eye by patching or applying drops to blur vision in the normal eye.

4-year-old boy is brought to the clinic due to crossed eyes. The left eye has been slightly deviated toward the nose over the last few months. Visual acuity testing reveals 20/20 and 20/50 in the right and left eyes, respectively. Corneal light reflex is central in the right eye and temporal in the left eye. When the right eye is covered the left eye shifts fixation from a nasal to a temporal direction. When the left eye is covered the right eye remains in the midline position. The red reflex is more intense in the left eye. Dilated funduscopic exam is normal. In addition to corrective lenses, which of the following is the best next step in management of this patient? A. Lantoprost drops to the left eye B. Lantoprost drops to the right eye C. No additional intervention indicated D. Patching of the left eye E. Patching of the right eye

E. Patching of the right eye Strabismus, or ocular misalignment, can be detected by asymmetric red & corneal light reflexes, as well as eye deviation on the cover test. Management includes correcting refractive error and strengthening the affected eye by patching or applying drops to blur vision in the normal eye.

4-year-old boy is brought to the clinic due to crossed eyes. The left eye has been slightly deviated toward the nose over the last few months. Visual acuity testing reveals 20/20 and 20/50 in the right eyes, respectively Corneal light reflexes central in the right eye & temporal in the left eye. When the right eye is covered the left eye shifts fixation from a nasal to a temporal direction. When the left eye is covered, the right eye remains in the midline position. The red reflex is more intense in the left eye. Dilated funduscopic exam is normal. In addition to corrective lenses, which of the following is the best next step in management of this patient? A. Latanoprost drops to the left eye B. Latanoprost drops to the right eye C. No additional intervention indicated D. Patching of the left eye E. Patching of the right eye

C. Lack of recommended immunizations Epiglottis is m/c due to Haemophilus influenzae type b and incompletely vaccinated individuals are at the highest risk. Manifestations include rapid onset of fever, sore throat, muffled voice, drooling, and stridor.

4-year-old boy is brought to the clinic due to sore throat. He first became ill 2 days ago with fever & rhinorrhea. This morning, he developed a severe sore throat and refused to eat his midmorning snack. Temp is 39.4C (103F). While sitting on the exam table, the patient appears distressed and leans forward with his chin thrusted forward. Tympanic membranes are clear. The tonsils are mildly erythematous and 1+ bilaterally. The submandibular space & neck are soft. Lung exam reveals no wheezes or crackles. Which of the following is the most likely risk factor for this patient's current condition? A. Childhood obesity B. Inadequate dental hygiene C. Lack of recommended immunizations D. Poorly controlled diabetes mellitus E. Secondhand smoke

C. Staphylococcus aureus This patient has findings consistent with bullous impetigo (bullae with yellow fluid aka "honey colored crust"). Impetigo is a superficial bacterial infection of the epidermis typically caused by bacterial invasion through a break in the skin barrier (wound, eczema, burn). Bullous impetigo is caused Staph aureus, which produces exfoliative toxin that disrupts cell adhesion in the superficial skin layers. Treatment is with oral ABX.

4-year-old girl is brought to the ED due to blisters on her left hand. 2 days ago, the patient developed small, fluid-like lesions, which have since increased in size and are now mild tender & itchy. She has a history of eczema and takes no meds. Temp is 38.1C (100.6F). Vitals are otherwise normal. Exam of the hand shows multiple flaccid bullae with yellow fluid and crusting. Which of the following is the most likely pathogen for this patient's condition? A. Coxsackievirus B. HSV C. Staphylococcus aureus D. Streptococcus pyogenes E. Varicella-zoster virus

C. inflammatory damage to the cochlea hearing loss is a common sequela of Strep pneumoniae meningitis due to inflammatory damage to the cochlea &/or labyrinth. Audiologic testing should be performed ASAP to identify hearing loss & improve outcomes if hearing rehab is needed

4-year-old girl is brought to the office due to behavioral concerns. Over the last 2 months, her parents & preschool teacher have noticed that she frequently does not respond to their questions & they often need to repeat themselves several times to get a response. 3 months ago, the patient was hospitalized for bacterial meningitis complicated by recurrent seizures. She has had no fever or HA since then & has not followed up until today's visit. VS & growth parameters are normal for age. In the office, the child is alert & can hop on 1 foot. She can copy a cross & square on paper. What is the most likely cause of this patient's current symptoms? A. bacterial invasion of the brain parenchyma B. global cerebral hypoxic-ischemic damage C. inflammatory damage to the cochlea D. recurrent 3 Hz-spike-and-wave epileptiform discharges E. thrombotic occlusion of dural venous sinus

D. Nitrofurantoin Glucose-6-phosphate dehydrogenase deficiency is an X-linked disorder associated with episodes of hemolytic anemia during oxidative stress. In order to prevent hemolysis, oxidative meds such Nitrofurantoin should be avoided.

4-year-old is brought to the office due to a day of dysuria & lower abdominal pain. The boy has been toilet-trained for a year but had an episode of urinary incontinence last night. The patient was hospitalized at age 2 for fatigue secondary to hemolytic anemia & was diagnosed with glucose-6-phosphate dehydrogenase deficiency. He has had no hematologic complications since then. PE shows mild suprapubic tenderness but no CVA tenderness. UA is positive for nitrites & leukocyte esterase. Which of the following ABX should be avoided in this patient? A. Amoxicillin-clavulanate B. Cefixime C. Cefpodoxime D. Cephalexin E. Nitrofurantoin

A. Abdominal US Infantile hypertrophic pyloric stenosis typically presents in a first-born male infant age 3-6 weeks with projectile vomiting immediately after feeding. Hypokalemic & hypochloremic metabolic alkalosis may be evident on lab eval, and abdominal US is diagnostic.

5-week-old boy is brought to the office by his mother for eval of vomiting. For the past 4 days, the boy has been vomiting forcefully right after drinking his formula. The patient has had fewer wet diapers today & had 1 dark brown stool yesterday. At age 2 weeks, the patient was at the 25th percentile weight & today is at the 10th percentile for weight. Temp is 36.7C (98.1F) and HR is 138/min. The lips & mucous membranes are slightly dry. The abdomen is soft & non distended with active bowel sounds. There is no hepatosplenomegaly or tenderness to palpation. Serum electrolyte results: Sodium: 133 mEq/L (normal: 136-145) Potassium: 3.1 mEq/L (normal: 3.5-5.0) Bicarbonate: 31 mEq/L (normal: 22-28) Chloride: 90 mEq/L (normal: 95-105) BUN: 20 mg/dL (normal: 7-18) Creatinine: 1.2 mg/dL (normal: 0.6-1.2) Which of the following studies will most likely confirm this patient's diagnosis? A. Abdominal US B. Air contrast enema C. Esophageal pH probe D. Fecal Hemoccult test E. Serum 21-hydroxylase level

B. Meningeal irritation Meningeal irritation often leads to pain or resistance to knee extension when the hip is flexed, also known as Kernig sign. Kernig sign is highly specific for meningitis & patients with a positive Kernig sign should undergo prompt lumbar puncture for CSF analysts

6-month-old boy is brought to the ED by his mother due to decreased feeding. Yesterday, he began to vomit breast milk. The patient was born at full term following an uneventful pregnancy. Vaccinations are up to date. Temp is 38.3C (101F). There is no skin rash. Pupils are equal and reactive and extraocular mov'ts intact. When the right hip is flexed, extension of the knee results in crying & flexion of the opposite knee. The abdomen is soft & no tender to palpation. Stroking the soles of both feet results in fanning of the toes with extension of the great toe. This patient's findings are suggestive of which of the following? A. Botulism B. Meningeal irritation C. Muscle inflammation D. Peritoneal inflammation E. Spinal cord compression

A. cholesteatoma cholesteatomas result from the accumulation of keratin debris & squamous epithelial cells within a TM retraction pocket. Sx include persistent otorrhea & conductive hearing loss & exam often reveals a pearly white mass behind an intact TM.

5-year-old boy brought to the clinic due to persistent purulent right ear discharge for a month, not relieved by topical or oral ABX. He has had no fever, ear pain, dizziness or tinnitus. The patient was born with a cleft palate that was prepared in infancy. Due to bilateral persistent middle ear effusions, tympanostomy tubes were also placed. 3 years ago, the tympanostomy tubes were removed, after which audiometry was normal. He has no other medical conditions. Temp 37.2C (99F). There is scant, yellow, malodorous insufflation & appears intact. There is retraction of the superior portion of the TM & a pearly white mass. The left TM is mobile & normal appearance. Audiometry reveals conductive hearing loss on the right side. Gait is normal. What is most likely responsible for this patient's presentation? A. cholesteatoma B. foreign body C. otitis externa D. otitis media E. vestibular schwannoma

A. arthrocentesis septic arthritis should be suspected in children with acute onset of fever & joint pain & swelling. Management includes arthrocentesis, empiric parenteral ABX & prompt surgical drainage

5-year-old boy is brought to ED due to fever & knee pain. The patient was diagnosed with Strep pharyngitis 2 weeks ago & completed a 10-day course of amoxicillin. 2 days ago, he developed fever & fatigue. Today he cries every time he bends his right knee & refuses to walk. Temp is 39.4C (102.9F). On exam, knee is swollen & tender to palpation. The patient holds knee in a flexed position & has pain with passive ROM. Lab results show: Leukocytes: 16,000/mm C-reactive protein: 62mg/L (normal < 8) ESR: 44mm/hr XR of knee shows joint effusion & soft tissue swelling. What is the next best step in management of this patient? A. arthrocentesis B. intraairticular CCS C. MRI of knee D. PCN E. rest & NSAIDs

A. Episodes of hemarthrosis Hemophilia is an X-linked recessive bleeding disorder that can present with recurrent hemarthrosis & skeletal muscle hematoma after minor trauma. Associated lab findings include prolonged PTT & decreased or absent Factor VII or IX activity

5-year-old boy is brought to the ED for eval of thigh pain. This morning, the patient bumped into the edge of a table in the living room & now has swelling & pain in his right thigh. Family Hx is significant for a maternal uncle who died after developing intracranial hemorrhages from a minor head injury last year. Exam shows an uncomfortable-appearing boy with marked ecchymoses & swelling over the right quadriceps. Which of the following will most likely be found on further history & exam of this patient? A. Episodes of hemarthrosis B. Old, healed rib fractures C. Pinpoint red spots on extremities D. Recurrent vasoocclusive crises E. Sister with heavy menstrual bleeding

B. Bone marrow eval Acute lymphoblastic leukemia is the most common cancer in children & presents with signs of bone marrow failure. The presence of >25% lymphoblastic in the bone marrow confirms the diagnosis.

5-year-old boy is brought to the ED for increasing fatigue. 3 weeks ago, he developed a fever & upper respiratory infection. The fever has persisted; he has had poor appetite & some weight loss. The patient has been less active & is refusing to walk. Temp is 39.4C (103F); BP: 100/70; HR: 140/min. Height & weight are at the 80th percentile. The oropharynx is clear & conjunctiva are pale. Enlarged, no tender lymph nodes are palpable in the cervical, auxiliary & inguinal regions. The spleen is enlarged. Palpation along bilateral femurs elicits tenderness. Multiple petechiae are seen. Lab results: Hematocrit: 20% (normal: 41-53%) Platelets: 18,000/mm3 (normal: 150,000-400,000) Leukocytes: 2,800/mm3 (normal: 4,500-11,000/mm3) CXR is normal. Which will confirm the diagnosis of this patient? A. Antinuclear antibody testing B. Bone marrow eval C. Epstein-Barr virus serology D. Hemoglobin electrophoresis E. HIV antibody testing

B. E. Coli Enterohemorrhagic E. Coli infection presents with bloody diarrhea & abdominal ain without fever. The most common subtype is E. Coli O157:H7 which produces Shiga toxins. Risk factors include consumption of undercooked beef & animal exposure. Antimicrobial use should be avoided to decrease the risk of hemolytic uremic syndrome.

5-year-old boy is brought to the ED with bloody diarrhea. Yesterday morning, the patient had loose, watery bowel mov'ts and nonbilious emesis, but his stool became grossly bloody as the day progressed. He has had 2 loose, bloody bowel mov'ts this morning, as well as abdominal pain. The patient went on a class trip to a local petting zoo a week ago. Temp: 36.6C (97.9F). Abdomen is not distended, but bowel sounds sound hyperactive. There is generalized abdominal tenderness without guarding or rebound. Rectal exam shows blood mixed with stool in the rectal vault. No petechial rashes are seen. What is the most likely cause of the patient's symptoms? A. Enterovirus B. E. Coli C. Giardia lamblia D. Norovirus E. Staph aureus

B. echo coarctation of the aorta is a congenital defect with symptoms that vary with patient age & severity of the narrowing of the aortic arch. Older infants & children are often asymptomatic & have the hallmark findings of > 10mmHg systolic BP differences between upper & lower extremities & diminished femoral pulses. Echo establishes the diagnosis & assesses for coexisting cardiac lesions.

5-year-old girl is brought in for routine visit. The girl was born full-term & has been healthy. Her immunizations are up to date. Weight & height have been tracking in the 50th percentiles. BP: 90/40 in right leg &90/45 in left leg, HR: 155/min, RR: 16/min. Pulse ox: 99% on room air. Exam shows well-appearing, cooperative girl with no dysmorphic features. Auscultation shows a 2/6 systolic murmur heard best at the left sternal border. Radial pulses are 2+ & femoral pulses are 1+. Which is the best next step in management of this patient? A. CXR B. echo C. indomethacin D. karyotype analysis E. propranolol

A. Acute rheumatic fever Acute rheumatic fever, a complication of streptococcal pharyngitis, is diagnosed clinically using the Jones criteria: fever, migratory polyarthritis, erythema marginatum & elevate acute-phase reactants (ESR, CRP).

5-year-old girl is brought to the clinic with joint pain & a rash. Last week, the patient developed pain in her knees. The pain resolved after a few days, but now her ankles & wrists are tender. She has also developed a nonpruritic, pink rash on her back. The patient had previously been healthy except for a sore throat a few weeks ago that resolved on its own. Temp is 38.3C (101F). The wrists & ankles are stiff and tender to manipulation. Multiple large, well-demarcated, erythematous, nonpruritic patches with a slightly raised outline are present on the trunk & proximal limbs. Lab results include- Leukocytes: 6,500/mm3 (normal: 4,500-11,000/mm3) Neutrophils: 56% (normal: 54-62%) Lymphocytes: 33% (normal: 25-33%) C-reactive protein: 32mg/L (normal: < 3 mg/L) ESR: 62 mm/h (normal: 0-20mm/h) Which of the following is the most likely diagnosis? A. Acute rheumatic fever B. Henoch-Schonlein Purpura C. Lyme Disease D. Systemic juvenile idiopathic arthritis E. Systemic lupus erythematosus

C. Personal history of intussusception The rotavirus vaccine effectively prevents most cases of rotavirus gastroenteritis. It is contraindicated in patients with a history of intussusception due to the small risk of this complication. Choice D- live virus vaccinations should no be administered to pregnant women due to the theoretical risk of fetal infection. However, live virus vaccinations can be safely administered to household contacts of pregnant women b/c transmission of vaccine-derived infection is exceedingly rare Choice E- Vaccines can be administered safely during minor illnesses (URI, otitis media). They should be postponed until recovery from moderate or severe illness (high fever) b/c manifestations of the underlying illness can be incorrectly attributed to the vaccine or vice versa.

6-month-old girl is brought to the office for a well-child visit & routine vaccinations. The patient is exclusively breastfed & is urinating & stooling normally. 1 month ago, she was hospitalized for intussusception that was reduced successfully by air enema. The patient lives in a small apartment with her mother, brother & maternal aunt who is pregnant. Vitals are normal. On exam, she has mild nasal congestion & clear rhinorrhea. The remainder of the exam is normal. Her mother is very concerned about the potential side effects of vaccinations. Which of the following is this patient's contraindication to the rotavirus vaccination? A. Administration of inactivated immunizations on the same day B. Family history of autism C. Personal history of intussusception D. Pregnant household member E. Viral upper respiratory infection

C.Retinoblastoma Leukocoria (white pupillary reflex) in an infant or young child should prompt immediate referral to an ophthalmologist to evaluate for retinoblastoma. Other presenting signs of retinoblastoma may include strabismus and nystagmus.

6-month-old girl is brought to the office for a well-child visit. The patient was born at 35 weeks gestation due to preterm labor. She is alert & interactive, smiling at the examiner & her mother. Ophthalmoscopic exam reveals a white reflex in the left eye and a red reflex in the right. Extraocular movements are intact, and the pupils are equal and reactive to light. The rest of the exam is normal. Which of the following is the most likely cause of this patient's physical exam findings? A. Bilateral cataracts B. Glaucoma C. Retinoblastoma D. Retinopathy of prematurity E. Strabismus

E. Seizure disorder This patient's LOC, gradual return to baseline, and transient hemiplegia are most consistent with a first-time seizure with Todd paralysis. Todd paralysis is a self-limited, focal weakness or paralysis that occurs after a focal or generalized seizure. It presents in the postictal period with hemiparesis or complete hemiplegia involving extremities on 1 side of the body. B/c Todd paralysis is a diagnosis from history alone, other causes of paralysis (embolism, intracranial bleed) should be ruled out (by CT or MRI) if there is no clear history of a preceding seizure. Treatment of Todd paralysis is supportive as the paralysis typically resolves within 36 hours.

6-year-old boy is brought to the ED due to difficulty moving his right arm and leg. He was playing in his room when his mother heard him fall. She found him unresponsive on the floor. The child regained consciousness after 3 mins; however, he was confused and unable to move his right side. The patient is otherwise healthy. Cranial nerves II-XII are intact. Heart sounds are normal. Right upper and lower extremities are flaccid and right-sided strength is 0/5. Left-sided strength in 5/5 in upper and lower extremities. Sensation to light touch is intact in all extremities. A CT scan and MRI of the brain are normal. The boy gradually regains complete motor function in his right arm and leg over the next 4 hours. Which of the following is the most likely cause of this patient's hemiplegia? A. Congenital heart disease B. Conversion disorder C. Hemiplegic migraine D. Intracranial hemorrhage E. Seizure disorder

B. Ethosuximide Absence seizures are a type of generalized epilepsy characterized by brief staring spells with momentary loss of awareness followed by an abrupt return to full consciousness. A classic 3-Hz spike wave is seen on electroencephalogram; 1st-line treatment is Ethosuximide. Valproate or Lamotrigine can also be used if 1st-line is not tolerated or is ineffective.

6-year-old boy is brought to the office by his mother due to difficulty with his behavior in school. His teacher is concerned that he does not pay attention in class & his grades are dropping. The patient's mother has noticed that at home he seems to stare into space for several seconds, multiple times throughout the day. This occurs even when he is watching his favorite tv show, and he seems to ignore her when she calls his name. The patient has no previous medical conditions & takes no meds. Which of the following is the most appropriate pharmacotherapy for this patient? A. Carbamazepine B. Ethosuximide C. Lorazepam D. Methylphenidate E. Phenobarbital

A. ASD ASD classically present with a wide, fixed, split S2 that does not vary with respiration & a mid systolic pulmonary flow murmur. Most of these defects close spontaneously by age 5

6-year-old boy is brought to the office for a well-child visit. His weight & height are at the 15th percentile for age & sex. BP: 94/52, HR: 92/min, RR: 16/min. Pulse ox is 99% on room air. Auscultation shows a normal S1 & a split S2 that does not vary with respirations. A 1/6 mid systolic murmur is heard. Which is the following is the most likely diagnosis for this patient? A. ASD B. coarctation of the aorta C. PDA D. tetralogy of Fallot E. VSD

B. Nontypeable Haemophilus influenzae Nontypeable Haemophilus influenzae, Streptococcus pneumoniae & Moraxella catarrhalis are the m/c bacterial causes of acute bacterial rhinosinusitis, which commonly presents with > 10 days of persistent nasal d/c & cough. In contrast, viral rhinosinusitis generally improves within 7-10 days. Fever may be present but is not required for diagnosis. Sinus tenderness is rare in young children but often found in adults. Patients with asthma may experience exacerbations (wheezing, coughing) triggered by concurrent URI.

6-year-old boy is brought to the office with a 10-day history of thick & persistent nasal d/c, nasal congestion & cough. He has had no associated vomiting, HA or earache. Temp is 37.2 C (98.9F), HR is 90/min & RR are 15/min. PE reveals yellow mucus dripping in the posterior nasopharynx. Tympanic membranes are clear bilaterally. Nasal turbinates are red & swollen. Maxillary sinuses are tender to palpation. The lungs are CTA bilaterally. Skin exam shows no rashes. Which of the following organisms is the most likely cause of this patient's condition? A. Aspergillus fumigatus B. Nontypeable Haemophilus influenzae C. Rhizopus arrhizus D. Staphylococcus aureus E. Streptococcus progenies

C. Spirometry Chronic cough (> 4 weeks) without specific historical or exam findings warrants spirometry testing to identify underlying obstructive lung disease (asthma).

6-year-old girl is brought to the clinic due to a prolonged cough. The patient started having rhinorrhea 6 weeks ago. 1 week later, she developed a cough with occasional expectoration of whitish sputum; the rhinorrhea resolved at that time. For the past 5 weeks, the patient's cough has continued throughout the day & night, occasionally awakening her & with no improvement. Her father tried using a humidifier, with no relief of her symptoms. The patient has been afebrile with no sick contacts. She takes no daily meds .Immunizations are up to date. The patient's mother has atopic dermatitis. Height & weight are in the 25th percentile. Temp is 36.7C (98F) and respirations are 24/min. PE is WNL. Which of the following is the best next step in eval of this patient? A. CT scan of chest B. PCR for Bordetella species C. Spirometry D. Sputum culture E. Sweat chloride test

C. Lower extremity hypotension This patient with right upper extremity HTN, lower extremity claudication & a heart murmur likely has aortic coarctation. The defect is usually congenital & located distal to the left subclavian. Although associated with Turner syndrome, aortic coarctation is most commonly sporadic & affects male patients. Key findings include upper HTN, lower extremity hypotension & weak/delayed distal pulses.

6-year-old is brought to the office by his mother for an annual PE. He has had intermittent pain in his legs during his soccer games for the past 3 months. The patient had no trauma preceding the onset of leg pain, has no numbness or tingling & is able to walk without a limp. Medical history is otherwise unremarkable. Temp is 36.1C (97F), BP is 135/88 in the right upper extremity, HR is 110/min, RR are 24/min and pulse ox is 98% on room air. Weight & height are at the 25th and 50th percentiles, respectively, for age. PE shows a well-appearing boy who has no dysmorphic features & who smiles & fully cooperates. A 2/6 continuous murmur is heard over the left inter scapular area. The lower extremities have no swelling, erythema or tenderness. Further eval would most likely show which of the following findings? A. Digital clubbing B. Hepatomegaly C. Lower extremities hypotension D. Parastenal heave E. Pulses paradoxus

D. schedule the patient for an orchiopexy Orchiopexy is the surgical fixation of the testis to the scrotal wall. Referral for surgery is indicated by age 6 months, as spontaneous testicular descent rarely occurs after this age. Orchiopexy is performed during infancy to optimize fertility & testicular growth & shown to decrease testicular torsion and may decreased (but not eliminate) the risk of testicular cancer.

7-month-old boy is brought to the office for a routine visit. The parents' only concern is that their son's "private area looks abnormal," which his primary care provider has been monitoring. The infant urinates frequently & stools once a day. Weight & length are at the 30th percentile. On PE, the penis is uncircumcised, & the urethral meatus is at the tip of the glans. The right testicle is palpated in the right hemiscrotum. The left scrotum is hypo plastic, hypopigmented & empty. A small mobile mass is palpable in the left inguinal canal. Which of the following is the best next step in management of this patient? A. Order a karyotype analysis B. Order an abdominal US C. Reassure & continue observation D. Schedule the patient for an orchiopexy E. Surgically excise the mass

C. Inpatient IV ABX Orbital cellulitis is a bacterial infection located posterior to the orbital serum & involves the muscles of extraocular mov't. Orbital cellulitis is a potentially life-threatening condition that requires inpatient IV ABX to prevent complications such as orbital abscess, cerebral abscess & cavernous sinus thrombosis

7-year-old boy brought to the ED due to worsening left eye pain for the past 2 days. He has no pain in his right eye. Temp is 39.2C (102.6F), BP: 102/70, HR: 94/min, RR: 18/min. PE shows mild proposes & opthalmoplegia of the left eye. The eyelid is erythematous, tender & swollen & the conjunctiva is hyperemic. The patient has pain with eye mov'ts. Visual acuity & pupillary reaction are normal bilaterally. Facial sensation is intact. CT scan reveals inflammation of extraocular muscles, fat stranding & mild anterior displacement of the globe. Which of the following is the best next step in management of this patient? A. Histopathy with culture of the orbit tissue B. Immediate surgical drainage of the orbit C. Inpatient IV ABX D. MRI Venmo graphs of the brain E. Outpatient oral ABX & follow up in 1 day

A. Desmopressin Desmopressin is the 1st-line pharmacotherapy for nocturnal enuresis. It can provide immediate improvement when behavioral modifications & alarm therapy have failed. When used as mono therapy, it has a high rate of relapse on discontinuation of therapy.

7-year-old boy is brought in due to persistent bed wetting. Since age 3, he has been able to stay dry during the day but continues to have "accidents" 3 nights a week. The child drinks fluids primarily in the morning & early afternoon & does not drink juice or caffeinated beverages. For the past 4 months, his parents have tried enuresis alarms & awarded "gold stars" for dry nights without improvement in the frequency of bed wetting. He has otherwise been in good health & has met all other developmental milestones. PE & UA are normal. Which of the following is the best next step in management of this patient? A. Desmopressin B. Imipramine C. Oxybutynin D. Serum glucose E. Vesicoureterogram

Enterobius vermicularis (pinworm) infection is highly contagious & manifests as nocturnal perinatal pruritus. 1st-line treatment is with Pyrantal pamoate or Albendazole

7-year-old boy is brought in for eval of anal pruritus. 1 month ago, while on a weekend camping trip with friends, he learned to swim in the lake & stayed in a log cabin. After returning home, the patient began to have increasing anal pruritus that is most severe at night. He has had no vomiting or diarrhea. VS are normal. Skin exam is normal. The lungs are CTA & abdomen is soft & nondistended. Inspection of the anus shows mild perinatal erythema with multiple excoriation but no induration, fluctuated or fissures. What is the most appropriate treatment for this patient? A. Chloroquine B. Metronidazole C. Nystatin D. Permethrin E. Pyrantel pamoate

B. Decreased platelet count HUS which is most commonly caused by a Shiga toxin-producing strain of E. Coli (serotype O157:H7) is characterized by microangiopathic hemolytic anemia, thrombocytopenia and acute kidney injury

7-year-old boy is brought to ED for fatigue. 1 week ago, he developed abdominal pain, vomiting & diarrhea. A few days later, he had significant blood in his stool, which has since resolved. The patient had been feeling better until today, when he had significant fatigue. Despite drinking a normal amount of fluids, the patient has not urinated in 24 hours. Multiple family members had similar initial GI symptoms after attending a family cookout last week, but they have all recovered. On PE, there is scleral icterus & 2+ pedal edema. A diagnosis of hemolytic uremic syndrome is suspected. Which of the following lab abnormalities is most likely associated with this patient's current condition? A. Decreased ferritin levels B. Decreased platelet count C. Decreased reticulocyte count D. Increased PT E. Positive Coombs test

C. Osteonecrosis Legg-Calve-Perthes disease (idiopathic osteonecrosis of the femoral epiphysis) classically presents in young children with progressive leg pain &/or a limp. Decreased hip ROM & thigh muscle atrophy may be present on exam & XR shows a flattened & fragmented femoral head.

7-year-old boy is brought to the office for eval of worsening chronic left thigh pain for 8 months. Recently, the patient began to limp. He has had no fever, additional joint pain or chronic medical conditions. While walking to the exam table, the patient avoids bearing full weight on the left leg & atrophy of the left proximal thigh muscle. The remainder of the exam is unremarkable. XR shows a flattened & fragmented femoral head with alternating regions of lucency & density. Which of the following is most likely cause of this patient's condition? A. Bone infection B. Malignancy C. Osteonecrosis D. Slipped epiphysis E. Stress fracture

A. Cystic fibrosis Pediatric chronic rhinosinusitis with nasal polyposis, especially when there are signs of exocrine pancreatic dysfunction (diarrhea & weight loss) should prompt eval for cystic fibrosis (sweat chloride testing)

7-year-old girl is brought to the clinic for eval of nasal congestion with facial pain. She first developed congestion over 1 year ago, which her parents have been treating with OTC allergy meds without improvement. Over the past 4 months, the congestion has worsened & now she has difficulty breathing through her nose. She developed a dry cough along with a constant, dull pain over her cheeks during the past week. Her parents have also noticed she has lost weight over the past 3 months. The girl reports that her stools have been loose. Temp is 37.8C (100F). Weight is <1st percentile at 18kg (39.7lb), down from 19kg (41.9lb) 3 months ago. PE reveals copious yellow mucus within both nares & translucent, grey, shiny masses that obscure the middle turbinate bilaterally. The lungs have coarse breath sounds bilaterally. What is the most likely diagnosis of this patient? A. Cystic fibrosis B. HIV infection C. Nasopharyngeal carcinoma D. Primary cilliary dyskinesia E. Seasonal allergic rhinitis

E. Smoking cessation for the mother Exposure to cigarette smoke is an important modifiable risk factor for acute otitis media. Cessation of smoking, continuation of breastfeeding, avoidance of day care & d/c of pacifier use should be encouraged before more aggressive action is taken.

8-month-old boy is brought to the office for follow-up of otitis media. The patient has had 2 episodes of acute otitis media in the past 3 months. 1 episode was in his left ear; the second was in his right ear 2 weeks ago. Both episodes resolved after a course of Amoxicillin. The patient has no other medical conditions & is meeting all growth & developmental milestones. He lives with his mother & 3-year-old brother, both of whom are in good health. The mother smokes a pack of cigarettes a day but only outdoors. The patient eats solid foods twice a day and breastfeeds every 3 hours, including overnight, when he falls asleep while feeding. On PE, the nares are clear & there is no pharyngeal erythema. The tympanic membranes are clear without erythema or effusion. The mother is concerned that the recurrent infections will affect his hearing & asks for advice on preventing future episodes. Which of the following is the most appropriate intervention in management of this patient? A. Cessation of overnight breastfeeding B. Immunization with pneumococcal polysaccharide vaccine 23 C. Initiation of daily ABX with amoxicillin D. Referral for bilateral tympanostomy tube placement E. Smoking cessation for the mother

RSV

8-month-old full-term boy brought to ED due to difficulty breathing. Sx began yesterday with "runny nose" and congestion. According to his parents, he has been breathing "faster than usual" today & is less interested in feeding. The patient received all vaccinations & takes no meds. Temp is 37.2 C (99F), HR: 124/min, RR: 50/min. Pulse ox: 92% on room air. Exam shows clear rhinorrhea, nasal flaring, grunting & retractions. Diffuse crackles & wheezes heard in both lungs. Which is most likely cause of infection? A. Bordetella pertussis B. mycoplasma pneumoniae C. parainfluenza virus D. RSV E. streptococcus pneumoniae

D. Methylphenidate

8-year-old boy brought in due to difficulty at school. The boy's teacher reports that he is disruptive to other students & his behavior was worsened over the course of the year. He has difficulty making friends due to his annoying behavior & trouble focusing. At home, he is active & affectionate but can easily become irritable & has trouble sharing toes with his younger siblings. His mother says, "he routinely forgets to do his homework despite being reminded multiple times & gets angry & argumentative when he is reprimanded." On exam, he is rigidity & interrupts the conversation with various questions & comments. What med would be most effective for this patient? A. Atomoxetine B. Clonidine C. Fluoxetine D. Methylphenidate E. Risperidone

E. rapid & deep breathing Ethylene glycol ingestion leads to a severe anion gap metabolic acidosis. This causes a typical rapid & deep breathing known as Kussmaul's respiration.

8-year-old boy is brought to ED by paramedics with a reported ingestion about 4 hours ago. The patient had an accidental ingestion of "antifreeze" after mistaking it for a beverage in the garage. Which exam fining is most likely to be found in this patient? A. dry skin & mucosal surfaces B. irregular HR C. excessive salivation D. pupillary dilation E. rapid & deep breathing

D. Topical Permethrin Scabies is a cutaneous mite infection that presents as an extremely pruritic, erythematous, papular rash. 1st line of non-crusted scabies is 5% topical Permethrin. Family members & household contacts should also be treated.

8-year-old boy is brought to the office with an itchy rash. 2 weeks earlier, he developed an erythematous rash on his arms, abdomen & groin area. The rash is intensely pruritic, particularly at night. 1 of his siblings has a similar rash. On exam, the boy has erythematous papular lesions on the extensor surfaces of his elbows & knees. Similar lesions are noted in his periumbilical area & on his buttocks. There are also several lesions on the flexor surfaces of his wrists & on his palmar surfaces. The papules are excoriated & some have hemorrhagic crusting. Which of the following is the most appropriate treatment for this patient's rash? A. Oral Prednisone B. Reassurance & observation C. Topical Hydrocortisone D. Topical Permethrin E. Topical Selenium Sulfide

E. Howell-Jolly bodies Howell-Jolly bodies are nuclear remnants of red blood cells that are usually removed by a functional spleen. Due to functional apple is in sickle cell patients, Howell-Jolly bodies are expected findings on peripheral smear

8-year-old is brought in for bilateral leg pain. His pain started yesterday & has gradually worsened. The patient has sickle cell disease and he has had 5 prior hospitalizations for similar pain episodes. His Hydroxyurea prescription has not been filled due to lack of insurance. Temp: 37.2C (99F). There is conjunctival pallor. Palpating of the bilateral legs shows tenderness but no erythema or swelling. CBC shows: Hemoglobin: 8.2g/dL (normal in boy's: 13.5-17.5) MCV: 84 fL (normal: 80-100) Reticulocytes: 8% (0.5-1.5%) Platelets: 380,000/mm3 (150,000-400,000) The patient's peripheral blood smear most likely has which of the following? A. Basophils stippling B. Bite cells C. Heinz bodies D. Helmet cells E. Howell-Jolly bodies

C. check carbamazepine levels non adherence to meds is a major cause of breakthrough seizures in patients with epilepsy. Prompt measurement of serum anti epileptic drug concentrations is an important part of the initial workup for these patients.

9-year-old boy brought to ED by his babysitter after having a seizure at home. The patient takes carbamazepine for epilepsy & has not had a seizure in years. It has been a stressful time for him as his parents are going through a divorce & he has been splitting time between 2 homes. The boy also just returned from a week-long stay at his grandparents' house. VS are normal. On PE, patient is drowsy but able to answer questions and follow directions. Pupils are equal & reactive to light. Cardiopulmonary exam is normal. The abdomen is soft & without hepatosplenomegaly. Muscle tone & strength are normal in all extremities. Which of the following is the best next step in evaluating this patient? A. check LFTs & ammonia levels B. evaluate for inborn errors of metabolism C. measure serum carbamazepine level D. order CT of head E. perform ECG

B. Bordetella pertussis Bordetella pertussis causes whooping cough, a vaccine-preventable disease characterized by severe coughing episodes, which are often associated with posttussive emesis & an inspiratory whoop. PE may reveal evidence of increased capillary pressure (facial petechiae, subconjunctival hemorrhage).

9-year-old boy is brought to the ED for a worsening cough. 2 weeks ago, he developed rhinorrhea, congestion & a mild cough. Since then, the cough has worsened, particularly at night & during exertion & he often vomits after a bout of coughing. Temp is 37.2C (99F), BP is 90/60, HR is 98/min and RR are 20/min. Pulse ox shows an oxygen saturation 98%. On PE, the child is alert and well-appearing. Bilateral subconjunctival hemorrhages & periorbital petechiae are noted. The lungs are CTA. Which of the following organisms is the most likely cause of this patient's illness? A. Adenovirus B. Bordetella pertussis C. Nontypeable Haemophilus influenzae D. Parainfluenza virus E. Respiratory syncytial virus (RSV)

C. Myocarditis Hand-foot-mouth disease (oral ulceration and extremity rash) & herpangina (oral ulcerations without rash) are the most common manifestations of enteroviral infection. Rare but serious complications of infection include myocarditis (heart failure, dysrhythmia, acute circulatory collapse) & central nervous system disease (rhombencephalitis, aseptic meningitis, acute flaccid paralysis).

9-year-old boy presents with throat pain & rash. 2 days ago, the patient was sent home from summer camp b/c he had fever, sore throat, pain with eating & a rash on his extremities. Vitals are normal. On PE, the conjunctivae & tympanic membranes are normal. Several superficial ulcers are present on the buccal mucosa & soft palate; the tonsils have no exudate. Skin exam findings show macular/vesicular exanthema on the hands, feet, buttocks, legs and arms. The infections agent most likely responsible for this patient's current condition is also associated with which of the following? A. Facial nerve palsy B. Keratitis C. Myocarditis D. Symmetric polyarthritis E. Temporal lobe encephalitis

D. Streptococcus pyogenes Acute rheumatic fever is a complication of untreated Streptococcus pyogenes pharyngitis. Major clinical features include carditis, chorea, erythema marginatum, SQ nodules & migratory arthritis.

9-year-old girl is brought to the ED due to sudden onset of uncontrollable, continuous writing of the arms & hands & grimacing of the face. 1 month ago, she had a sore throat & fever that resolved after a few days without treatment. Her vaccinations are current. Vitals are normal. On exam, the oropharynx has no exudate or tonsillar enlargement. Cardiac exam reveals a pericardial friction rub. Erythrocyte sedimentation rate (ESR) is 65 mm/hr. ECG shows prolonged PR interval & diffuse ST elevations. Infection with which of the following organisms is the most likely cause of this patient's symptoms? A. Coxsackievirus B. Parvovirus B19 C. Streptococcus mutans D. Streptococcus pyogenes E. Trypanosoma cruzi

D. Reassurance & observation only A cephalohematoma is a collection of blood between the skull & periosteum caused by the rupture of subperiosteal vessels during delivery. Exam reveals a firm, non fluctuant scalp swelling that does not cross suture lines. Management is with reassurance & observation b/c most cephalohematomas resolve without intervention.

A 1-day old girl is being evaluated in the newborn nursery. She was born at 39 weeks gestation. Vitals are normal. Exam shows a vigorous newborn with a firm, non-fluctuant scalp swelling confined to the right side of the skull. In addition to monitoring bilirubin levels, which of the following is the best next step in management? A. Brain MRI B. Cranial molding helmet C. Hematoma evacuation D. Reassurance & observation only E. Serial hemoglobin levels

C. gonococcal conjunctivitis typically presents in the first 2-5 days of life with eyelid swelling, copious & purulent eye drainage & conjunctival injection. Treatment includes inpatient management & a single dose of Ceftriaxone or Cefotaxime. If left untreated, patients with gonococcal conjunctivitis can develop blindness.

A 3-day-old girl is brought to the ED due to copious, purulent eye drainage for the past day. She was born vaginally to a 31-year-old woman, G3P3. The family declined routine prophylactic meds, including vitamin K, erythromycin ointment & hepatitis B vaccination, for the newborn. The patient was d/c with her parents at 24 hours of life. She is tolerating breastfeeding with normal voiding & stooling. Temp: 36.8C (98.2F). Exam shows bilateral eyelid edema, conjunctival injection & purulent discharge. The remainder of PE is unremarkable. What is the most likely diagnosis in this patient? A. chlamydial conjunctivitis B. dacryostenosis C. gonococcal conjunctivitis D. HSV conjunctivitis E. Staph aureus conjunctivitis

E. Reassurance & gentle handling Birth weight > 4kg, shoulder dystocia & vacuum delivery are risk factors for neonatal clavicular fractures. Management includes reassurance & gentle handling, as most neonatal clavicular fractures heal rapidly without complications.

A 5-hour-old girl is evaluated for decreased movement of the left arm. She was born at 40 weeks gestation by vacuum-assisted vaginal delivery to a mother with gestational DM. The delivery was complicated by shoulder dystocia. Birth weight was 4.6 g (10lb). Palpation of the left clavicular bone reveals crepitus. Passive movement of the left upper extremity elicits crying. Moro reflex is decreased of the left. Bilateral extremities are neurovascularly intact. Which of the following is the best next step in management of this patient? A. Administer aspirin for analgesia B. Closed reduction using traction C. Figure-of-8 splint placement D. Open reduction & internal fixation E. Reassurance & gentle handling

D. Nutritional rickets Nutritional rickets classically occurs in exclusively breastfed infants without vitamin D supplementation. Presentation includes craniotables (soft skull bones) and forearm deformities (widened wrists, radial/ulnar bowing). Lower extremity bowing occurs once patients are weight-bearing.

A 6-month-old boy is brought to the office for a well-child visit. The patient drinks 24-28oz daily of donated, pasteurized breast milk. He was adopted from Uganda at age 2 months. Immunizations are up to date. He has no chronic medical conditions & takes no meds or supplements. Height & weight are at the 20th & 40th percentiles, respectively. Head circumference is at the 30th percentile. The anterior fontanel is open & flat, and the skull bones are soft & flexible to pressure. Bilateral swelling of the wrist is present. Cardiopulmonary & abdominal exams are normal. There are no skin rashes. An XR of the wrist shows metaphysical widening with cupping & fraying. Which of the following is the most likely cause of the patient's XR abnormality? A. Congenital syphilis B. HIV C. Nonaccidental trauma D. Nutritional rickets E. Systemic juvenile idiopathic arthritis

B. fibrosis of renal interstitial space Vesicoureteral reflux is a risk factor for recurrent UTI & if untreated, can lead to renal scar formation, HTN & chronic renal insufficiency. Scarring is focal due to chronic interstitial inflammation & fibrosis.

A 6-month-old boy is brought to the office for follow-up after a UTI. The boy was initially brought in 2 weeks ago with fever for 4 days. A urine culture at that 1st visit grew > 100,00 colony forming units of E. coli. The fever resolved within 48 hours with ABX & a full course of ABX was completed. Renal US performed 2 days ago showed mild right hydronephrosis; voiding cystourethrogram reveal dilation of the right ureter with blunting for the right renal calyces. The patient has no other medical conditions. Temp: 36.7C (98F). PE is normal. If untreated which is the most likely long-term complication of this patient's condition? A. development of renal cysts B. fibrosis of renal interstitial space C. inflammation of the renal glomerulus D. no long-term complications E. obstruction from uric acid stones

C. Administer a measles-containing vaccine today & again at age 4 The measles vaccine is administered at age 12-15 months & 4-6 years. It should also be administered to infants age 6-11 months who travel internationally or to an outbreak area but is less likely to result in long-term protection. Therefore, these patient should still receive the standard 2-dose series.

A healthy 12-month-old boy is brought to the office for a WCC. He has been growing & developing well. The patient received a dose of measles-containing vaccine at age 6 months prior to visiting the Philippines. His PE is normal. Which of the following is the best approach to protecting this child from measles? A. Administer a measles-containing vaccine at age 4 only B. Administer a measles-containing vaccine today only C. Administer a measles-containing vaccine today & again at age 4 D. Administer the measles vaccine as a separate injection from all other live-virus vaccines E. Draw antibody tigers & administer a measles-containing vaccine if tigers are inadequate

B. Azithromycin Group A streptococcal pharyngitis typically presents with sore throat, fever, tonsillar exudates & tender anterior cervical LAD. 1st-line treatment is PCN or amoxicillin. Patients with a Hx of anaphylaxis to PCN should be treated with Azithromycin or Clindamycin.

A previously healthy 14-year-old boy is brought to the office with 2 days of fever, sore throat, pain with swallowing & abdominal pain. He has a history of anaphylaxis after receiving PCN. Temp is 38.9C (102F), BP is 110/60, HR is 94/min and RR are 14/min. Exam shows posterior oropharynx erythema & bilateral tonsillar exudates. Several bilateral, tender, enlarged lymph nodes are palpable in the anterior cervical region. Monospot test is negative. Rapid streptococcal antigen testing is positive. Which of the following is the most appropriate treatment for this patient? A. Amoxicillin B. Azithromycin C. Cephalexin D. Ciprofloxacin E. Trimethoprim-sulfamethoxazole

D. Reye Syndrome Patients with Reye Syndrome present with vomiting, encephalopathy, hepatic dysfunction & abnormal behavior leading to seizures & lethargy. Caregivers should avoid administering aspirin to children with viral infections

A previously healthy 2-year-old girl is in the ICU due to coma & respiratory failure. The patient began having a headache & confusion a day ago & started vomiting this morning. The patient began having generalized tonic-clonic seizures in the ED & was intubated & sedated. The patient had fever, cough & pharyngitis last week. The nanny gave the child OTC meds for fever but the parents don't know what it was. Temp is 37.1C (98.8F). Pupils are equal & have sluggish reaction to light. The sclera is nonicteric & the neck is supple. Mild hepatomegaly is palpated on abdominal exam. Lab results: LFTS- total bilirubin: 0.5mg/dL (normal: 0.1-1.0) AST: 906 U/L (normal: 8-20) ALT: 1214 U/L (normal: 8-20) CSF- opening pressure: 320mm of H2O (70-180) glucose: 53 mg/dL (normal: 40-70) protein: 32mg/dL (< 40) leukocytes: 2/mm3 (normal: 0-5) CT scan of the brain reveals diffuse cerebral edema. What is the most likely diagnosis? A. anticholinergic toxicity B. bacterial meningitis C. Hep A infection D. Reye syndrome E. viral encephalitis

D. Mumps virus Among patients with meningitis, a viral etiology is suggested when CSF analysis reveals a mildly elevated WBC count & protein level & normal glucose concentration. Mumps virus is capable of causing viral meningitis in unvaccinated individuals. Other classic symptoms of mumps include parotitis (swelling around the mandible in this case) & orchitis.

An 18-month-old boy is brought to the ED due to a fever. Yesterday, he developed a fever, ashiness & fatigue; today he has vomited 3 times. The patient has not yet received the routine 12- to 15-month vaccines. Temp is 39.4C (102.9F). PE reveals an irritable child who cries & closes his eyes during funduscopic exam. When the examiner flexes the neck, the child flexes the knees. There is swelling around the right mandible. The oropharynx has no erythema or exudate. DTRs are 2+. CSF analysis results are as follows- Glucose: 65 mg/dL (normal: 40-70) Protein: 75 mg/dL (normal: < 40mg/dL) Leukocytes: 100/mm3 (normal: 0-5 cells/mm3) RBCs: 1/mm3 (normal: 0-5 cells/mm3) Which of the following is the most likely cause of this patient's illness? A. Cryptococcus neoformans B. haemophilus influenzae type B C. Listeria monocytogenes D. Mumps virus E. Streptococcus pneumoniae

D. Type 2 pneumocytes Respiratory distress syndrome occurs when type 2 pneumocytes produce insufficient amounts of surfactant, curing increased alveolar surface tension and decreased lung compliance. Surfactant deficiency is common in premature infants but can also occur in infants of diabetic mothers.

Newborn baby is admitted to the neonatal hospital ICU due to respiratory distress. He was born to a 34-year-old primigravida whose pregnancy was complicated by DM1. The boy was born via C-section after a prolonged labor. Weight is 10lbs 6oz. PE immediately after delivery shows tachypnea, nasal flaring and intercostal retractions. Pulse ox is 78% on room air. The infant is intubated & mechanical ventilation is started. High pressures are required to achieve normal tidal volume and keep alveoli inflated. This infants condition is caused by dysfunction of which of the following? A. Fibroblasts B. Macrophages C. Type 1 pneumocytes D. Type 2 pneumocytes E. Vascular smooth muscle

D. observation only a congenital umbilical hernia is typically soft & reducible & does not require intervention. These hernias usually close spontaneously by age 5.

a 4-hour-old boy is evaluated in the nursery for routine newborn care. The patient was born at 38 weeks gestation via spontaneous vaginal delivery to an 18-year-old woman who did not receive prenatal care. Birth weight is 3.6kg (8lb). VS are normal. Exam shows a sleeping infant in no distress. There is molding of the occiput. A soft mass inferior to the umbilical stump is 1.4cm in diameter & covered by skin. The mass increases in size when the infant cries & reduces into the abdominal cavity with gentle pressure. The mass does not seem tender to palpation. The rest of PE is normal. What is the next best step in management? A. abdominal US B. immediate surgery C. karyotype analysis D. observation only E. topical silver nitrate


Kaugnay na mga set ng pag-aaral

Microbiology (Dr. AL, LSUE) Exam 2

View Set

Mod 5 Intro to Info Sec WGU ~ Cryptography

View Set

Comprehensive AANP Review (LEIK/Barkley/Fitzgerald) pt. 3 (printed from TiffenyWade)

View Set

Chapter 4 in Political Science Research Methods

View Set

Exam like 5000 Accounting 2 Houke TNU

View Set